Дефект матрицы: Ранг, дефект линейного оператора.

Содержание

Дефекты и неисправности матриц

Матрица – не только хрупкая, но и достаточно дорогая часть ноутбука. При этом разбитая матрица ремонту не подлежит, ее необходимо менять на новую. Но если проблемы, которые замечены на мониторе возникли из-за какого-либо другого элемента экрана, то неприятность может быть устранена недорогим ремонтом. Рассмотрим наиболее распространенные дефекты и неисправности матриц ноутбуков.

При обнаружении дефектов изображения или его отсутствия, в первую очередь следует убедиться, что причина действительно в нерабочем экране. Проверить это не всегда просто. Один из наиболее надежных и доступных вариантов вариантов – это подключение к ноутбуку внешнего монитора. Если внешний монитор не отобразит возникших дефектов, значит проблема именно в экране ноутбука, в противном случае причины неисправности следует искать совсем в другом месте.

Но если проблема все-таки в экране, то нужно определить, что именно не работает.

Из чего состоит экран

Экран состоит из матрицы, системы подсветки, управляющей схемы, инвертора (в ноутбуках с флуоресцентной подсветкой) и шлейфа. Основой изображения является подсветка – именно она даёт свечение пикселей. Она может быть или светодиодной, или ламповой, в таком случае для свечения лампы ещё необходим инвертор и питание к нему. Свет от ламп или светодиодов проходит через матрицу и за счёт этого формируется изображение. Сигнал для этого формирования идёт по шлейфу и проходит через плату, которая преобразует его в удобный для матрицы формат.

Рассмотрим неисправности каждой из составляющих экрана.

Дефекты матрицы

Неисправности матрицы ноутбука обычно имеют физический характер, так как сама матрица по своей сути не является сложным электронным устройством. Именно поэтому большинство из них довольно легко заметить.

Если на поверхности матрицы невооруженным глазом можно заметить чёрные подтёки из жидких кристаллов, а само изображение будет отсутствовать либо будут видны лишь отдельные его фрагменты, то матрица физически повреждена. При наиболее сильных повреждениях на поверхности будут хорошо заметны трещины и сколы с потёками.

Физически поврежденная матрица восстановлению не подлежит, её просто меняют на новую. В магазине «Коннект» вы можете не только заказать и купить необходимую матрицу, но и произвести ее замену.

Дефектом матрицы можно считать битые пиксели. Это светящиеся определённым светом или полностью тёмные точки на экране.

Кстати! Битые пиксели могут присутствовать даже на новой матрице, поэтому при покупке ноутбука следует внимательно просмотреть монитор в различных режимах. Наличие битых пикселей в малых количествах не считают неисправностью. Так что если через месяц или два после приобретения ноутбука, вы заметили один битый пиксель, то можете не спешить воспользоваться гарантией. Скорей всего это не получится. Другое дело, если таких дефектов будет много.

Со временем они могут появляться на старых матрицах и иногда их можно заставить работать. К примеру, если пиксель светиться одним светом, его можно восстановить специальными программами, тёмные пиксели восстановлению не подлежат.

Неисправности подсветки и инвертора

Характерным признаком неисправности подсветки является отсутствие изображения на экране. На самом деле оно есть, просто очень тусклое, и если присмотреться, его можно заметить при достаточно хорошем внешнем освещении. Иногда лампы могут садиться в течение некоторого времени, что выражается в значительном снижении яркости картинки и появлению красноватого оттенка по краям экрана.

Похожие признаки возникают и при поломке инвертора, только в этом случае экран ещё может произвольно выключаться или включаться.

У светодиодной подсветки проблема проявляется несколько иначе. Выход из строя одного или нескольких светодиодов не приведёт к поломке всей подсветки, а проявится в некоторых областях экрана. Если же не работает весь массив, то проблема, скорее всего, кроется в неисправных элементах управляющей схемы.

Несмотря на то, что подсветка является частью матрицы, её можно заменить или отремонтировать. Сама процедура ремонта достаточно тонкая и сложная, поэтому лучше всего её проводить в сервисном центре.

Неисправности и дефекты шлейфа

Так как шлейф является связующим звеном между аппаратной частью ноутбука и экраном, любое его повреждение приводит к искажению или отсутствию изображения. Сильные перегибы или повреждения отдельных контактов приводят к появлению цветных вертикальных или горизонтальных полос, а повреждение наиболее важных контактов может вообще проявляется в виде неработоспособного экрана.

Плохой контакт или перегиб шлейфа можно распознать по открытию или закрытию крышки ноутбука. Если дефекты появляются в момент этого действия,  скорее всего проблема именно в шлейфе.

Замена шлейфа не является сложной процедурой, в большинстве случаев её можно провести дома. Но если же проблема в контактах на платах, то лучше обратиться за заменой или ремонтом к специалистам.

Неисправности управляющей схемы

Внешние признаки поломки управляющей схемы могут быть самыми разнообразными, поэтому диагностирование подобных неисправностей непростая задача. Внешне поломка одного из элементов может привести как к неработоспособности экрана, так и к появлению визуальных дефектов: цветные полосы, рябь, инвертированные цвета. В таком случае поможет лишь ремонт в сервисном центре – нерабочий элемент необходимо выпаять и на его место установить новую деталь.

Все поломки экрана имеют свои характерные особенности, зачастую связанные тем, какую роль они играют в выводе информации на экран. Но некоторые неисправности видеокарты и материнской платы могут привести к схожим внешним признакам, поэтому лучше всего заранее проверить их работоспособность, например, как рекомендовалось выше — подключив внешний монитор.


Клим Вопрошалов

Дефекты и неисправности матриц ноутбуков

Матрица – это самая хрупкая и достаточно дорогая часть ноутбука, поэтому при падениях и ударах или при неаккуратном обращении она просто может выйти из строя. Разбитая матрица ремонту не подлежит, ее необходимо менять на новую. А вот если проблемы возникли с каким-либо другим элементом экрана, всё может ограничиться недорогим ремонтом.

Если у Вас появились дефекты изображения или оно вовсе отсутствует, надо убедиться, что причина в нерабочем экране. Проверить это не всегда легко. Один из простейших вариантов –подключить к ноутбуку внешний монитор. Если такой возможности нет, то можно ориентироваться по звукам – если динамики работают и слышно звук загрузки системы, то проблема вероятно в нерабочей матрице. В ином случае, скорее всего, неисправна видеокарта или компонент материнской платы (чаще всего виноваты мосты).

Итак, если проблема в экране, то нужно определить, что не работает.

Экран состоит из матрицы, системы подсветки, управляющей схемы, инвертора (в ноутбуках с флуоресцентной подсветкой) и шлейфа. Основой изображения является подсветка – именно она даёт свечение пикселей. Она может быть или светодиодной, или ламповой, в таком случае для свечения лампы ещё необходим инвертор и питание к нему. Свет от ламп или светодиодов проходит через матрицу и за счёт этого формируется изображение. Сигнал для этого формирования идёт по шлейфу и проходит через плату, которая преобразует его в удобный для матрицы формат.

Ниже мы вкратце рассмотрим неисправности каждой составляющей экрана и детально обговорим внешние проявления этих поломок.

Дефекты матрицы

Итак, неисправности самой матрицы могут быть, в основном, физического характера, так как сама матрица по своей сути не является сложным электронным устройством. Именно поэтому большинство из них довольно легко заметить.

Если на поверхности матрицы невооруженным глазом можно заметить чёрные подтёки из жидких кристаллов, а само изображение либо будет отсутствовать, либо будут видны лишь отдельные его фрагменты, то матрица повреждена или разбита. При сильных повреждениях на поверхности будут хорошо заметны трещины и сколы с потёками.

Дефектом матрицы можно считать битые пиксели. Это светящиеся определённым светом или полностью тёмные точки на экране. Они могут присутствовать даже на новой матрице, и их наличие в малых количествах не является неисправностью. Со временем они могут появляться на старых матрицах и иногда их можно заставить работать. К примеру, если пиксель светиться одним светом, его можно восстановить специальными программами, тёмные пиксели восстановлению не подлежат.

Физически поврежденная матрица восстановлению не подлежит, её просто меняют на новую. Заказать и купить матрицу для ноутбука можно в нашем магазине.

Неисправности подсветки и инвертора

Характерным признаком неисправности подсветки является отсутствие изображения на экране. На самом деле оно есть, просто очень тусклое, и если присмотреться, его можно заметить при достаточно хорошем внешнем освещении. Иногда лампы могут садиться в течение некоторого времени, что выражается в значительном снижении яркости картинки и появлению красноватого оттенка по краям экрана.

Похожие признаки возникают и при поломке инвертора, только в этом случае экран ещё может произвольно выключаться или включаться.

У светодиодной подсветки проблема проявляется немного по-другому. Выход одного или нескольких светодиодов не приведёт к поломке всей подсветки, просто некоторые области экрана будут плохо подсвечиваться. Если же вышел весь массив, то проблема, скорее всего, кроется в неисправных элементах управляющей схемы.

Несмотря на то, что подсветка является частью матрицы, её можно заменить или отремонтировать. Сама процедура ремонта достаточно тонкая и сложная, поэтому лучше всего её проводить в сервисном центре.

Неисправности и дефекты шлейфа

Так как шлейф является связующим звеном между аппаратной частью ноутбука и экраном, любое его повреждение приводит к искажению или отсутствию изображения. Сильные перегибы или повреждения отдельных контактов приводят к появлению цветных вертикальных или горизонтальных полос, а повреждение важных контактов может вообще проявляется в виде неработоспособного экрана.

Плохой контакт или перегиб шлейфа можно распознать по открытию или закрытию крышки ноутбука. Если в момент этого действия изображение меняется, появляются дефекты – скорее всего проблема в самом шлейфе.

Замена шлейфа – простая процедура, в большинстве случаев её можно провести дома, если же проблема в самих контактах на платах – то лучше доверить их замену или ремонт специалистам.

Неисправности управляющей схемы

Внешние признаки поломки управляющей схемы могут быть самыми разнообразными, поэтому диагностирование подобных неисправностей непростая задача. Внешне поломка одного из элементов может привести как к неработоспособности экрана, так и к появлению визуальных дефектов, таких как цветные полосы, рябь, инвертированные цвета. В таком случае поможет лишь ремонт в сервисном центре – нерабочий элемент просто выпаивается и на его место ставиться новая деталь, при серьезных поломках матрица меняется целиком.

Таким образом, мы видим, что все поломки экрана имеют свои характерные особенности, зачастую связанные тем, какую роль они играют в выводе информации на экран. Нужно помнить, что некоторые неисправности видеокарты и материнской платы могут привести к схожим внешним признакам, поэтому лучше всего заранее проверить их работоспособность, например, подключив внешний монитор.

Замену матрицы или ремонт компонентов экрана мы рекомендуем Вам доверить профессионалам – при самостоятельной разборке или замене можно легко повредить шлейфы, матрицу или корпус ноутбука, что в свою очередь приведёт к более дорогому ремонту ноутбука.

Как обнаружить дефекты матрицы ноутбука в домашних условиях

Главная » Блог » Полезные советы

Полезные советы

Автор Кравцов Даниил На чтение 3 мин Просмотров 1.5к. Опубликовано

При пропадании изображения на экране ноутбука или появлении его искажений не следует сразу же думать о самом плохом – замене дорогой матрицы. Причин неисправности может быть много – например, износ шлейфа или нарушения в работе видеокарты. Поэтому самое главное – это определить причину плохой работы.

Проще и надежнее всего отказаться от пустой траты времени на самостоятельные поиски и обратиться к специалистам в сервисный центр. Тем не менее, есть неисправности, которые можно диагностировать своими силами, не имея опыта в ремонте такого рода аппаратуры.

Закажи профессиональный ремонт ноутбука в Москве или продай его на запчасти!
Бесплатная диагностика и выезд курьера. Срочный ремонт. Гарантия. Недорого!
Звони! Тел.: +7 (903) 729-32-48 или напиши нам на email: [email protected]

Заказать

Содержание

  1. Дефекты матрицы и ее окружения
  2. Локализация дефекта
  3. Проверка шлейфа
  4. Неужели матрица
  5. Типичные дефекты изображения
  6. Темное изображение
  7. Искажения
  8. Однотонный экран

Дефекты матрицы и ее окружения

Проверка матрицы ноутбука

Матрица ноутбука – это жидкокристаллический экран, подсвечиваемый набором светодиодов или лампочек, который питается через инвертор – специальный трансформатор. Она является одним из самых дорогих комплектующих ноутбука. Неисправности в матрице обычно возникают в результате неаккуратного обращения, а также механического воздействия острыми или тяжелыми предметами. Случаются и производственный брак или неисправности в системе электропитания. Каким же образом можно проверить исправность матрицы и определить, что именно она является причиной неработоспособности или некачественной работы всего ноутбука?

Локализация дефекта

Устройство следует соединить с работоспособным монитором. Если изображение будет хорошим, то к процессору и видеокарте претензий нет. Остается сама матрица со вспомогательными элементами, а также шлейф. Возможны и другие варианты, однако они встречаются крайне редко.

Проверка шлейфа

Следует несколько раз открыть и закрыть крышку ноутбука, следя при этом за качеством изображения – не возникают ли на нем при этом какие-либо дефекты.

Неужели матрица

Следующим этапом можно проверить саму матрицу на механические нагрузки – изгиб и скручивание. Необходимо иметь в виду, что подвергать ее таким воздействиям следует с чрезвычайной осторожностью. Если же небольшие усилия отражаются на качестве изображения, экран требуется заменить.

Типичные дефекты изображения

Дефекты экрана ноутбука

Независимо от марки и модели ноутбука наиболее часто встречаются одни и те же неисправности.

Темное изображение

Изображение становится очень темным. Разница между включенным и выключенным устройством почти незаметна. К такому дефекту обычно приводит поломка инвертора, который обеспечивает питание подсветки экрана. Чаще всего он выходит из строя не мгновенно – сначала экран мерцает, а во время работы ноутбука слышен дополнительный шум. Инвертор можно заменить, обратившись к специалистам.

Искажения

Возникновение искажений в виде вертикальных цветных полос может быть следствием неисправности матрицы или шлейфа. Конкретизировать в этом случае источник неисправности можно в сервисном центре. Также в связи с высокой стоимостью матрицы нет смысла, не имея опыта, заменять ее самостоятельно.

Однотонный экран

Серый или черный экран без признаков изображения, как правило, является следствием механического дефекта, возникшего в результате сильного удара или падения.

При этом необратимые изменения возникают в матрице, и она нуждается в замене. Следует провести проверку работоспособности видеокарты, подключив ноутбук к монитору.

Выше были приведены некоторые дефекты, которые могут появиться на экране устройства и потребовать вмешательства специалистов. Чаще всего замена неисправных деталей не требует специальных знаний.

Выкупим твой бу ноутбук в любом состоянии не старше 2010 года!
Быстрая оценка по телефону, бесплатные выезд курьера по Москве. Деньги — сразу.
Звони! Тел.: +7 (903) 729-32-48 или напиши нам на email: [email protected]

Продать ноутбук

Поделиться с друзьями

Оцените автора

( 1 оценка, среднее 5 из 5 )

Полосы на экране ноутбука — неприятная неожиданность 💻

По сравнению с персональными компьютерами, на ноутбуках несколько сложнее диагностировать причину неисправности, т. к. почти вся конструкция – это одно целое. Очень неприятная проблема – это полосы и пятна на экране ноутбука.

Вроде бы, и обращаешься с ноутбуком бережно, и пользуешься им не постоянно, а проблема всё-равно появляется. Что же делать?

Содержание

Причины появления дефектов на экране

На экранах обычных мониторов и ноутбуков полосы могут появиться по нескольким причинам:

  • Выход из строя матрицы, поможет только замена
  • Повреждение, излом, плохой контакт шлейфа от матрицы к плате ноутбука
  • Неисправная видеокарта. В некоторых случаях возможен ремонт или замена видеокарты
  • Неисправность материнской платы, как правило из-за перегрева компонентов

Прогноз неутешительный, практически во всех случаях требуется ремонт в сервисном центре. В домашних условиях можно лишь заменить видеокарту, если она не встроенная, или удалить и переустановить драйвер. Отдельные «левши» и «кулибины» могут попытаться заменить шлейф или матрицу самостоятельно, но стоит учесть ряд нюансов, о которых речь пойдёт ниже.  Давайте сначала попробуем на глаз определить истинную причину неисправности ноутбука.

1. Для первоначальной диагностики причины появления полос или пятен нам потребуется обычный монитор, который нужно подключить к ноутбуку по любому интерфейсу: VGA, HDMI, Display Port. Если на мониторе изображение отличное, без полос и артефактов, значит проблема в матрице или шлейфе. Если и на мониторе есть полосы, тогда нужен ремонт/замена видеокарты или материнской платы ноутбука.

2. Проверяем реакцию матрицы на физическое воздействие: пробуем двумя руками, как бы, изогнуть и выкрутить экран, типа как выкручивают тряпки, пробуем слегка надавить пальцем на поверхность экрана. Естественно без фанатизма. Если полосы видоизменяются, значит неисправна матрица или шлейф, что менее вероятно.

3. Смотрим, пропадают полоски, или может меняются, при открывании и закрывании крышки ноута. Пробуем медленно и быстро. Изменения в любую сторону свидетельствуют о неисправности шлейфа матрицы.

4. Полоса может быть рядом битых пикселей, особенно если она разноцветная. Это совершенно другая проблема, изучите пожалуйста отдельную статью.

Далее постараемся визуально определить что именно нуждается в ремонте. Вы можете посмотреть видео и продолжить чтение статьи:


Посмотреть это видео на YouTube

Матрица ноутбука требует замены

Если вертикальные полосы появляются или исчезают при изгибе или кручении экрана, а на внешнем мониторе всё нормально

Если работает только одна сторона экрана, а другая полностью белая

Или с другими артефактами, но явно видно, что работает только часть дисплея

Белая широкая полоса или большое пятно в любой части матрицы

На экране характерная рябь

Разноцветные полосы на экране

При изгибе матрицы изображение размывается

Если все цвета отображаются в негативе, т.е. белый – чёрным, а чёрный белым и т.д. На картинке логотип «TOSHIBA» должен быть красным, а он – зелёный, а фон не чёрный, а светло-серый

Замена или ремонт шлейфа матрицы

Требуется если на экране много разных цветных линий или изображение мерцает и рябит, но при движении крышки ноутбука картина меняется. При этом внешний монитор работает без нареканий.

Шлейф дешевле и проще всего заменить, считайте, что вам повезло.

Требуется ремонт видеокарты или материнской платы

Если на экране разноцветные линии, горизонтальные либо вертикальные, как правило рассредоточенные по всему экрану, и на внешнем мониторе такая же картина:

Рябь по всему экрану, но замена матрицы или шлейфа не приводит к положительному результату

Визуально почти не возможно определить, что сбоит именно видеокарта, на 100% здесь поможет только другой монитор.

В случае с поломанным шлейфом есть три выхода

  1. Отдать в ремонт
  2. Попытаться найти на рынке или в интернете точно такой же, для этой модели матрицы, но это маловероятно, ввиду широкого разброса наименований. В сервис-центре вам вряд ли продадут такой шлейф. Лучше подождут пока вы набегаетесь и сдадите ноутбук в ремонт.
  3. Можно попробовать отремонтировать старый шлейф. Для этого понадобятся паяльник, обычный тестер, провод типа МГТФ и навыки электронщика, или знакомый с этими навыками 🙂 Суть ремонта заключается в поиске оборванного проводника шлейфа с помощью тестера, и его замене.
    Новый проводок МГТФ нужно прокинуть сверху старого и припаять с обоих концов.

Провод МГТФ состоит из одного или нескольких проводников, изолированных друг от друга. Изоляция провода МГТФ состоит из незапекаемой фторопластовой пленки.

Можно использовать обычные тонкие и прочные медные провода, просто МГТФ самые распространённые. Продаются на радиорынках и в магазинах, например в «Чип и Дип».

Что делать с неисправной видеокартой или материнской платой

В компьютерах и ноутбуках видеокарты бывают нескольких видов. В ноутбуках видеокарты обычно встроены (интегрированы) в процессор, или дискретные, т.е. распаянные на плате. Это зависит от модели ноутбука. Раньше внешние видеокарты вставлялись в PCMCIA слот и её можно поменять даже самостоятельно. Правда стоят они недёшево, но не так дорого как ремонт в сервисном центре.

В случаях неисправной встроенной видеокарты или материнской платы, ноутбук придётся отдавать в ремонт. Удовольствие не из дешёвых, поэтому требуйте согласования цены ремонта ОБЯЗАТЕЛЬНО!

Цена ремонта может быть как за новый ноутбук, но даже при половинной стоимости задумайтесь, стоит ли оно того? Ремонтный ноутбук может подвести ещё раз, причём в самый неподходящий момент. Почитайте как выбрать себе новый ноутбук.

При неисправности видеочипа вам могут предложить три варианта лечения:

  1. Прогрев чипа. Самый дешёвый вариант, нет гарантии что поможет, плюс неисправность может проявиться снова.
  2. Ребоулинг чипа. Дороже, но при исправной микросхеме даёт 99% удачного исхода событий.
  3. Замена чипа. 99.9% успеха, но намного дороже чем ребоулинг, и не факт, что замена чипа необходима.

Ребоулинг (с англ. Перешаровка) – полный демонтаж и повторный монтаж чипа, по-простому это просто перепайка чипа, но называется по-другому из-за особенностей монтажа. Микросхемы BGA крепятся к плате с помощью шариков из припоя.

Материнская плата могла повредиться в следствии перегрева, или из-за окислившихся контактов, или могла слегка поржаветь из-за разлитого кофе. Варианты ремонта такие же, как и в случае с ремонтом видеокарты, плюс дополнительные меры.

Решать вам, в зависимости от стоимости ноутбука, ремонта и рекомендаций мастера. Чтобы иметь по-меньше проблем советую не покупать б/у ноутбуки! Если у вас есть замечания или дополнение к статье, оставляйте комментарий, обсудим!

Поднижнечелюстной лимфатический узел крысы после введения в мандибулярный костный дефект матрицы из полигидроксиалканоата с мультипотентными мезенхимальными стромальными клетками

Несмотря на развитие травматологии и ортопедии, полное восстановление костной ткани является проблемным, поскольку большие дефекты не могут заживать спонтанно. Использование стволовых клеток — все более расширяющаяся область исследования, дающая надежду на успех терапевтических методов лечения ран и травм, на которые невозможно эффективно воздействовать современными хирургическими методами [5, 7].

Красный костный мозг содержит прогениторные клетки (плюрипотентные стромальные — ПСК), способные к дифференцировке в костную, хрящевую и другие виды соединительной ткани. Это позволяет широко применять такие клетки для ускорения регенерации костей [1, 5].

В научной литературе имеется множество данных об эффективности использования клеточных технологий в стоматологии, травматологии и хирургии. Однако полностью отсутствуют результаты исследования лимфатических узлов после указанных способов воздействия на регенерацию костной ткани, тогда как именно эти органы являются маркером выраженности местного воспалительного процесса, по изменениям в них можно точно оценивать результативность тех или иных лечебных мероприятий, предсказывать развитие многих осложнений, а значит, и успешно принимать меры по их профилактике.

В связи с вышеизложенным цель исследования — изучение реакций регионарных лимфатических узлов, обусловленных имплантацией аутологичных мультипотентных мезенхимальных стромальных клеток костномозгового происхождения (АММСККП) для ускорения регенерации дефекта кости нижней челюсти в эксперименте.

Материал и методы

Работа основана на результатах морфологического исследования особенностей изменения структурной организации субмандибулярных лимфатических узлов крыс-самцов инбредной линии Wag в разные сроки при введении в искусственно созданный дефект кости угла нижней челюсти АММСККП, адсорбированных на матрице из полигидроксиалканоата (ПГА).

Эксперименты проводили на самцах крыс инбредной линии Wag весом 180-200 г возрастом 6 мес. Все манипуляции с животными осуществляли под общим ингаляционным эфирным наркозом в условиях чистой операционной с соблюдением «Правил проведения работ с использованием экспериментальных животных» (Приказ МЗ СССР №755 от 12 августа 1977 г.; Приказ Министерства высшего и среднего специального образования СССР №742 от 13 ноября 1984 г.). На каждую точку исследования было использовано не менее 6 крыс (от 6 до 12 особей, всего 62 животных).

АММСККП выделяли, вымывая костный мозг из эпифизов бедренных костей у крыс-самцов линии Wag. Полученную суспензию клеток помещали в пластиковые флаконы («Nunk», Дания), через 48 ч после эксплантации костного мозга неприкрепившиеся клетки сливали. Прикрепившиеся клетки культивировали в среде α-МЕМ с добавлением 10% эмбриональной телячьей сыворотки («Biolot», Россия) при 37 °С в СО2 инкубаторе с 5% СО2 в условиях насыщенной влажности. Смену среды производили каждые три дня. При субкультивировании монослойную культуру рассевали в плотности 1000-5000 клеток/см2 (в зависимости от ростовых свойств используемой эмбриональной сыворотки), использовали стандартные растворы Версена и трипсина. Физические, морфологические, фенотипические признаки и дифференцировочный потенциал полученных клеток соответствовали таковым для АММСККП, которые были определены и описаны нами ранее [3, 4].

АММСККП 2 пассажа, полученные от крысы указанной линии, трансфицировали ДНК плазмиды pЕGFP-N1 (Clontech Laboratories Inc., USA), содержащей ген зеленого флюоресцентного белка GFP под контролем промотора цитомегаловируса и ген устойчивости к неомицину под контролем промотора вируса SV40, необходимого для последующей селекции с использованием дженетицина G418 (pEGFP-N1; «Clontech Laboratories Inc.», USA). Подробный протокол трансфекции и способы оценки экспрессии введенного гена GFP в АММСККП изложены в наших более ранних работах [4].

ПГА (сополимер из 85% полигидроксибутирата и 15% гидроксивалериата) в виде матриксов холодного прессования (авторское название) диаметром и высотой 2 мм был предоставлен для исследования Институтом биофизики СО РАН (Красноярск). Полимер до адсорбции АММСККП стерилизовали замоченными в забуференном физиологическом растворе для культур клеток в автоклаве при 120 °С, давлении в 1 атмосферу в течение 20 мин. Непосредственно перед внедрением приготовленный для имплантации фрагмент ПГА для пассивной адсорбции клеток погружали в суспензию АММСККП в культуральной среде (1·106 клеток в 1 мл суспензии) на 2 ч, в связи с тем, что живые клетки прикрепляются к любому твердому субстрату.

Модель дефекта костной ткани в эксперименте: под общим ингаляционным эфирным наркозом, в условиях чистой операционной, при соблюдении правил асептики и антисептики, после обработки кожи спиртом скальпелем производили разрез кожи длиной 1,5-2 см по нижнему краю нижней челюсти. Тупым способом при помощи распатора отслаивали жевательную мышцу и обнажали поверхность кости нижней челюсти в области ее угла. Стоматологическим бором делали круглое отверстие диаметром 2 мм в кости угла нижней челюсти, с полостью рта дефект кости не сообщался. После внедрения ПГА с адсорбированными АММСККП или без клеток (контроль) послойно ушивали рану викрилом. Животные с признаками гнойно-воспалительных осложнений (абсцессы в подкожной клетчатке) из эксперимента выбраковывались и в дальнейших исследованиях не участвовали.

Спустя 1, 2, 3, 4 и 5 нед после операции субмандибулярные (регионарные к месту применения АММСККП) лимфатические узлы фиксировали в 4% растворе параформальдегида на фосфатном буфере (рН 7,4) не менее 24 ч, обезвоживали в серии этанола возрастающей концентрации, просветляли в ксилоле и заключали в парафин. Неокрашенные срезы толщиной 5-7 мкм изучали на световом микроскопе Axioimager M1 («Carl Zeiss», Германия) при увеличении до 1200 раз в режиме люминесценции с фильтром Alexa 488.

Результаты исследования

Процессы репарации искусственно созданного дефекта костной ткани в области угла нижней челюсти крыс как без какого-либо воздействия, так и на фоне имплантации ПГА без АММСККП описаны нами ранее [2, 3].

Статистически достоверные отличия между состоянием центров размножения субмандибулярных лимфатических узлов после имплантации в участок повреждения кости нижней челюсти чистого ПГА или ПГА с АММСККП не найдены.

Вместе с этим через 1 нед после имплантации ПГА с адсорбированными АММСККП в дефект нижнечелюстной кости в регионарных (субмандибулярных) лимфатических узлах при исследовании в отраженном ультрафиолетовом свете были найдены овальные скопления ярко светящихся клеток. Эти клетки были очень крупными, до 20 мкм в диаметре, и такие клетки располагались в лимфоидных узелках. Во флюоресцирующих клетках светилась не вся цитоплазма, а разные по размерам овальные гранулы, т.е. клетка представляла собой скопление ярко светящихся частиц с четкими ровными краями. Размер этих гранул достигал 10 мкм. Иногда было видно темное овальное ядро (рис. 1, а, б, см. на цв. вклейке).Рисунок 1. Рис. 1а. Субмандибулярные лимфатические узлы крыс в различное время после имплантации ПГА с адсорбированными АММСККП с трансфицированным геном GFP в дефект кости нижней челюсти. Неокрашенные срезы в отраженном ультрафиолетовом свете с фильтром Alexa 488. Через 1 нед после операции в корковом веществе присутствуют овальные скопления ярко светящихся крупных клеток.Рисунок 1. Рис. 1б. Субмандибулярные лимфатические узлы крыс в различное время после имплантации ПГА с адсорбированными АММСККП с трансфицированным геном GFP в дефект кости нижней челюсти. Неокрашенные срезы в отраженном ультрафиолетовом свете с фильтром Alexa 488. Фрагмент рис. 1, а. Светящиеся клетки в овальном скоплении достигают в размере 20 мкм, в их цитоплазме содержатся разные по размерам флюоресцирующие гранулы размером до 10 мкм, иногда видно темное овальное ядро.

Спустя 2 нед после применения ПГА с АММСККП для воздействия на репарацию поврежденной нижней челюсти в лимфатических узлах количество крупных специфически светящихся клеток увеличилось. Причем увеличилось как число овальных скоплений таких объектов, так и число клеток в них. По-прежнему в таких клетках светилась не вся цитоплазма, а множество различных по размерам гранул (см. рис. 1, в, г, на цв. вклейке).Рисунок 1. Рис. 1в. Субмандибулярные лимфатические узлы крыс в различное время после имплантации ПГА с адсорбированными АММСККП с трансфицированным геном GFP в дефект кости нижней челюсти. Неокрашенные срезы в отраженном ультрафиолетовом свете с фильтром Alexa 488. Обширные скопления из крупных специфически светящихся клеток расположены в паренхиме коркового вещества спустя 2 нед после хирургического вмешательства.Рисунок 1. Рис. 1г. Субмандибулярные лимфатические узлы крыс в различное время после имплантации ПГА с адсорбированными АММСККП с трансфицированным геном GFP в дефект кости нижней челюсти. Неокрашенные срезы в отраженном ультрафиолетовом свете с фильтром Alexa 488. Фрагмент рис. 1, в. Большое число крупных клеток с различными по размерам флюоресцирующими гранулами в овальном скоплении, ткань которого отличается по плотности и фоновой окраске от окружающих тканей и имеет четкую границу. Кроме того, ярко светятся эритроциты во внутриузловых капиллярах.

К следующему сроку применения, к 3-й неделе, площадь скоплений крупных светящихся клеток значительно уменьшилась, также сократилось и количество клеточных элементов в них. Но вместе с этим резко возросло число флюоресцирующих клеток, расположенных поодиночке в структурах лимфатических узлов (см. рис. 1, д, е на цв. вклейке).Рисунок 1. Рис. 1д. Субмандибулярные лимфатические узлы крыс в различное время после имплантации ПГА с адсорбированными АММСККП с трансфицированным геном GFP в дефект кости нижней челюсти. Неокрашенные срезы в отраженном ультрафиолетовом свете с фильтром Alexa 488. К 3-й неделе после имплантации площадь скоплений крупных светящихся клеток значительно уменьшилась, но вместе с этим резко возросло число крупных флюоресцирующих объектов, расположенных поодиночке в паренхиме органов.Рисунок 1. Рис. 1е. Субмандибулярные лимфатические узлы крыс в различное время после имплантации ПГА с адсорбированными АММСККП с трансфицированным геном GFP в дефект кости нижней челюсти. Неокрашенные срезы в отраженном ультрафиолетовом свете с фильтром Alexa 488. Фрагмент рис. 1, в. Количество крупных светящихся клеток в скоплениях стало меньше.

Этот процесс сокращения размеров скоплений светящихся клеток и числа самих объектов со свечением в них прогрессировал и на 4-й и 5-й неделях. К этому времени можно отметить не равномерное расположение крупных светящихся клеток в лимфоидных узелках, а выстраивание таких объектов по периферии узелков. Также следует обратить внимание на постепенное уменьшение количества светящихся гранул в указанных крупных клетках (см. рис. 1, ж, з на цв. вклейке).Рисунок 1. Рис. 1ж. Субмандибулярные лимфатические узлы крыс в различное время после имплантации ПГА с адсорбированными АММСККП с трансфицированным геном GFP в дефект кости нижней челюсти. Неокрашенные срезы в отраженном ультрафиолетовом свете с фильтром Alexa 488. На 4-й неделе в паренхиме коркового вещества расположены единичные скопления светящихся клеток. В этих структурах только по периферии содержится небольшое число крупных клеток с различными по размерам флюоресцирующими гранулами. Рисунок 1. Рис. 1з. Субмандибулярные лимфатические узлы крыс в различное время после имплантации ПГА с адсорбированными АММСККП с трансфицированным геном GFP в дефект кости нижней челюсти. Неокрашенные срезы в отраженном ультрафиолетовом свете с фильтром Alexa 488. Через 5 нед после операции единичные крупные светящиеся клетки расположены только на периферии овальных структур, количество флюоресцирующих гранул в таких клетках стало меньше.

На рисунках с большим увеличением (см. рис. 1, б, г, е-з на цв. вклейке) четко видно, что ткань овальных скоплений светящихся клеток отличается по плотности и фоновой окраске от окружающих тканей и имеет четкую границу. Это дает возможность для предположения о концентрации клеток со светящейся цитоплазмой в лимфоидных узелках.

Кроме этого, на некоторых из указанных рисунков (см. рис. 1, г, е-з на цв. вклейке) хорошо видна флюоресценция эритроцитов в сосудах, проходящих рядом со скоплением крупных светящихся клеток и внутри его. Эритроциты, согласно литературным данным, обладают аутофлюоресценцией [10].

В лимфатических узлах животных после применения ПГА без АММСККП светящиеся объекты отсутствовали как в лимфоидных узелках, так и в других структурах данных органов (рис. 2, а-е, см. на цв. вклейке).Рисунок 2. Рис. 2а. Поднижнечелюстные лимфатические узлы в разные сроки после внедрения ПГА без АММСККП в участок повреждения нижнечелюстной кости в эксперименте. Неокрашенные срезы в отраженном ультрафиолетовом свете с фильтром Alexa 488. Через 1 нед после имплантации видны единичные небольшие светящиеся объекты (клетки), являющиеся эритроцитами во внутриузловых сосудах.Рисунок 2. Рис. 2б. Поднижнечелюстные лимфатические узлы в разные сроки после внедрения ПГА без АММСККП в участок повреждения нижнечелюстной кости в эксперименте. Неокрашенные срезы в отраженном ультрафиолетовом свете с фильтром Alexa 488. Спустя 2 нед после хирургического вмешательства присутствуют единичные небольшие флюоресцирующие объекты. Рисунок 2. Рис. 2в. Поднижнечелюстные лимфатические узлы в разные сроки после внедрения ПГА без АММСККП в участок повреждения нижнечелюстной кости в эксперименте. Неокрашенные срезы в отраженном ультрафиолетовом свете с фильтром Alexa 488. Светящиеся объекты практически отсутствуют на 3-й неделе после операции.Рисунок 2. Рис. 2г. Поднижнечелюстные лимфатические узлы в разные сроки после внедрения ПГА без АММСККП в участок повреждения нижнечелюстной кости в эксперименте. Неокрашенные срезы в отраженном ультрафиолетовом свете с фильтром Alexa 488. Фрагмент рис. 1, в. Все флюоресцирующие клетки являются эритроцитами, расположенными во внутриузловых сосудах.Рисунок 2. Рис. 2д. Поднижнечелюстные лимфатические узлы в разные сроки после внедрения ПГА без АММСККП в участок повреждения нижнечелюстной кости в эксперименте. Неокрашенные срезы в отраженном ультрафиолетовом свете с фильтром Alexa 488. К 4-й неделе после имплантации светятся только эритроциты в капиллярах и сосудах.Рисунок 2. Рис. 2е. Поднижнечелюстные лимфатические узлы в разные сроки после внедрения ПГА без АММСККП в участок повреждения нижнечелюстной кости в эксперименте. Неокрашенные срезы в отраженном ультрафиолетовом свете с фильтром Alexa 488. Флюоресценция эритроцитов во внутриузловых крупных сосудах и капиллярах крысы через 5 нед после операции. В широком поле зрения были видны единичные небольшие светящиеся объекты (клетки), но это были обладающие аутофлюоресценцией эритроциты во внутриузловых сосудах (см. рис. 2, а-е на цв. вклейке) [10].

Обсуждение

В течение 5 нед после имплантации ПГА с адсорбированными АММСККП с трансфицированным геном GFP в дефект кости нижней челюсти в субмандибулярных лимфатических узлах присутствовали овальные скопления ярко светящихся клеток. Эти клетки были очень крупными, до 20 мкм в диаметре (см. рис. 1, а-з на цв. вклейке).

Такие клетки с флюоресценцией располагались в лимфоидных узелках, в пользу чего свидетельствует то, что на некоторых препаратах четко видны различия в плотности тканей между скоплениями светящихся клеток и вокруг них (см. рис. 1, б, г, е-з на цв. вклейке) и то, что такие скопления имеют шаровидную форму и расположены только в корковом веществе недалеко от капсулы, т.е. в корковом плато, а не в паракортексе и не среди мозговых синусов в мякотных тяжах.

Скорее всего светящиеся клетки в лимфатических узлах являются макрофагами. Такое заключение сделано на основании нескольких причин:

1. Размер клеток. Только очень немногие клетки могут превышать по размеру 20 мкм, и среди таких клеточных элементов — макрофаги.

2. Наличие множества разнокалиберных включений, которые, скорее всего, являются лизосомами.

3. Неправильная форма клеток.

4. Расположение в лимфатических узлах. В лимфатических узлах макрофагов очень много в герминативных центрах лимфоидных узелков, куда они представляют антигены для осуществления иммунных функций и где они фагоцитируют и лизируют клетки с признаками деструктивных изменений, множество которых образуется при делении и дифференцировке В-лимфоцитов.

В таком случае светящиеся разнокалиберные гранулы в макрофагах являются лизосомами с поглощенным флюоресцентным материалом.

Необходимо отметить, что имеется множество данных об аутофлюоресценции макрофагов в некоторых условиях [6]. Но в таком случае должна быть точно такая флюоресценция макрофагов в лимфатических узлах животных после применения ПГА без АММСККП. Однако в данном случае в лимфатических узлах светятся только небольшие клетки (эритроциты), а скопления крупных светящихся объектов полностью отсутствуют (см. рис. 2, а-е на цв. вклейке).

Можно сделать предположение, что такие крупные светящиеся клетки в лимфатических узлах являются не просто макрофагами, а макрофагами, которые фагоцитировали введенные АММСККП или структуры, сформированные из таких клеток. В научной литературе имеются результаты исследований, указывающие на возможность флюоресценции макрофагов за счет свечения фагоцитированного и метаболизированного материала в их гранулах [8].

В месте повреждения кости нижней челюсти с последующей имплантацией ПГА с адсорбированными АММСККП развивается асептическая воспалительная реакция, характеризующаяся поступлением в регионарные лимфатические узлы, в данном случае — субмандибулярные, большого объема клеточного и тканевого детрита, который поглощается макрофагами.

Несомненно, что вместе с этим детритом в лимфатические узлы поступает и часть введенных АММСККП.

В узлах макрофаги поглощают антигенный материал и, видимо, попавшие туда фрагменты АММСККП вместе со светящимся белком GFP. В таких случаях лизосомы макрофагов, в которых находится флюоресцентный белок, могут светиться в отраженном свете так же, как и сами АММСККП с трансфицированным геном GFP. Также не исключено и встраивание ДНК белка GFP в геном макрофагов и наработка этого белка, но такая возможность кажется маловероятной.

То, что макрофаги с поглощенным клеточным светящимся детритом сконцентрированы в лимфатических узлах в герминативных центрах узелков, можно объяснить с 2 позиций.

Во-первых, в герминативных центрах идут размножение, активация и дифференцировка клеток В-линии, которые в дальнейшем будут синтезировать антитела против определенных антигенов. Для запуска и успешного осуществления этого процесса необходимо присутствие макрофагов с данным антигеном. В самих центрах размножения имеется множество макрофагов, которые могут поглощать антигены из окружающих тканей и, видимо, лимфы, проходящей по промежуточным синусам.

Во-вторых, макрофаги могут фагоцитировать светящиеся клетки или вышедший из них при разрушении белок GFP из лимфы и затем или мигрировать в узелки для активации и запуска процесса пролиферации и дифференцировки В-лимфоцитов или передавать антигены другим макрофагам, находящимся уже непосредственно в лимфоидных узелках. Существует возможность фагоцитирования АММСККП и их детрита в месте имплантации ПГА, миграции макрофагов с антигеном в лимфатические узлы вместе с током лимфы и уже в этих органах миграции или передачи антигенов фагоцитам центров размножения.

Таким образом, не исключено создание иммунной защиты против введенных АММСККП с трансфицированным геном GFP или самого светящегося белка. Не исключено, что после повторного введения клеток с трансфицированным геном GFP такие клеточные элементы будут очень быстро уничтожаться системой иммунитета и из них не будет формироваться никаких структур.

Необходимо рассмотреть еще одну возможную причину свечения макрофагов. Известно, что эритроциты обладают выраженной аутофлюоресценцией [10]. При моделировании повреждения кости нижней челюсти повреждаются многочисленные кровеносные сосуды и эритроциты оказываются в тканях.

В месте геморрагий клетки крови поглощаются макрофагами (сидерофагами), кроме того, эритроциты из тканей поступают в регионарные лимфатические узлы, где также фагоцитируются макрофагами. Такое присутствие эритроцитов и продуктов их распада в лизосомах макрофагов может обусловить их флюоресценцию под воздействием ультрафиолетового облучения. В литературе есть данные, описывающие флюоресценцию макрофагов вследствие наличия в них гемосидерина [9].

Однако при повреждении кости нижней челюсти и имплантации ПГА без АММСККП в тканях также оказывается множество эритроцитов, которые затем также фагоцитируются макрофагами и попадают в лимфатические узлы. Но при исследовании лимфатических узлов животных этой группы скопления светящихся макрофагов в лимфоидных узелках не были обнаружены.

Количество крупных светящихся клеток увеличивалось до 2-й недели, причем увеличилось как число овальных скоплений таких объектов, так и содержание клеток в них. По-прежнему в таких клетках светилась не вся цитоплазма, а множество различных по размерам гранул (см. рис. 1, а-г на цв. вклейке).

Начиная с 3-й недели содержание светящихся клеточных элементов начало снижаться. На 4-5-й неделях в лимфоидных узелках присутствовали только единичные клетки со свечением, которые были выстроены по периферии узелков. К окончанию времени наблюдения также можно отметить постепенное уменьшение количества светящихся гранул в таких макрофагах (см. рис. 1, д-з на цв. вклейке).

Постепенно введенные АММСККП замещаются собственными клетками и структурами. Вследствие того, что структур, построенных из имплантированных АММСККП, становится все меньше, в лимфатических узлах сокращается количество светящихся макрофагов, содержание светящихся гранул в них и интенсивность свечения.

Заключение

После внедрения в участок повреждения кости нижней челюсти ПГА с адсорбированными АММСККП с трансфицированным геном GFP в лимфоидных узелках регионарных лимфатических узлов появляются многочисленные крупные макрофаги с множеством овальных светящихся включений в цитоплазме. Численность таких макрофагов нарастает в течение 2 нед после операции, а далее начинает уменьшаться. Видимо, введенные таким способом АММСККП частично поглощаются макрофагами. При разрушении структур, сформированных из АММСККП, детрит также фагоцитируется макрофагами. В том и другом случае эти макрофаги оказываются в герминативных центрах лимфоидных узелков лимфатических узлов, где не исключена инициация иммунитета против ДНК и белка GFP.

В Canon 750D и 760D обнаружился дефект матрицы

?

Прошлый пост | Следующий пост

Ряд западных источников сообщает, что специалисты фотопроката LensRentals.com в первых партиях Canon 750D и 760D обнаружили серьёзный дефект матриц.

В толще стекла, покрывающего сенсор, обнаружились какие-то включения, напоминающие пыль. При съёмке на открытой диафрагме на изображение это не влияет, и видимой проблема становится только около f/11 и при бОльшем закрытии диафрагмы, как и с обычной пылью. На снимке дефект выглядит довольно специфично:

По информации фотопроката, они уже сообщили в Canon о проблеме и специалисты вендора серьёзно озаботились ситуацией. Рекомендую почитать о ситуации в первоисточнике.

  • Примеры снимков с Fujifilm X-h2 (RAW)

    Желающие могут скачать и у себя покрутить равы с Fujifilm X-h2. Напоминаю, что для этого нужен Lightroom версии 7.2 либо Adobe Camera RAW 10.2. Если…

  • 38 мне уже, вам поэтому обзор

    Кто-как отмечает ДР, а я встретил его за монтажным столом, собирая первый обзор новой Sony A7 Mark III на 24-мегапиксельном сенсоре с 693 фазовыми…

  • Я — человек-штатив?

    Двести лет не был в ЖЖ, как-то руки сейчас тексты строчить не доходят — с грехом пополам добираюсь до инстаграма ( @Q3D2), зато стал чаще…

  • Тёплый плед, уютное кожаное кресло, полумрак и новогодние огни полагаются

    Всю неделю занят новогодними делами: фотографируем в рождественских огнях, морозим фотомоделей в метель, проводим воркшопы по съёмке зимней красоты.

  • Первый пошёл: новая версия конвертера Capture One Pro

    Вот и поддержка нового E-M1 Mark II появляться начала: ORF (RAW) этой камеры без танцев с бубном умеет открывать выпущенный на прошлой неделе…

  • Суперхайрез: как выглядит результат в 80 мегапикселей

    Фото новенькой BMW i8 без ретуши, как было из Lightroom, с лёгким подтягиванием пересветов. В исходном варианте RAW имеет разрешение 80.7…

  • Как они интересно с полем резкости работают

    Увидел вчера ссылку на этот мультик у Алекса Экслера, не могу не поделиться: https://vk.com/video-53258521_456242262…

  • Спрашивавшим про ISO 6400? Ход котом.

    Это Шелдон, кот(омодель) Родиона Газманова, и не просто так, а в ISO 6400 при 1/60s и f/1. 8 на 150 миллиметрах…

  • Второй раз в парке развлечений для взрослых мальчиков

    Думал сначала сделать цветной фоторепортаж, как и в прошлый раз, но… в итоге остановился на монохроме, потому что цвет теперь роли не играл…

Апрель 2020
ВсПнВтСрЧтПтСб
   1234
567891011
12131415161718
19202122232425
2627282930  

  • live13 : (без темы) [+1]
  • rezanoe_gorlo : (без темы) [+1]
  • timon_timonich : (без темы) [+1]
  • ivandalavia : (без темы) [+0]
  • ivandalavia : (без темы) [+5]
  • emilles : (без темы) [+0]

Разработано LiveJournal. com

linear алгебра — Интуитивно понятно, почему у дефектных матриц есть лишние собственные значения?

Спросил

Изменено 7 месяцев назад

Просмотрено 834 раза

$\begingroup$

Недавно я видел пример того, как сдвиг $$\begin{bmatrix} 1 & 1\\ 0 & 1\end{bmatrix}$$ является примером дефектной матрицы, так как она имеет собственные значения $1,1 $, но только один независимый собственный вектор $\mathbf{v}_1 = (1,0)$.

Итак, в этом случае я вижу, что алгебраическая кратность больше, чем геометрическая кратность. Но мне интересно почему система создала «два» собственных значения, когда только одно из них было «фактически собственным значением» — ( Дело даже не в том, что было два собственных вектора, каждый из которых соответствовал $1$, $1$).

Есть ли основная причина, по которой система выдала два собственных значения? (Если бы я интуитивно догадался, что произойдет, чисто с геометрической точки зрения, я бы предположил, что характеристический многочлен будет просто линейным: $\lambda — 1$. Хотя это невозможно для $2 \times 2$, Мне было интересно, есть ли какое-то другое значение для второго собственного значения?)

  • линейная алгебра
  • матрицы
  • собственные значения-собственные векторы
  • линейные преобразования
  • интуиция

$\endgroup$

1

$\begingroup$

Ну, я бы сказал, что есть только одно собственное значение: $1$. Дело в том, что мы обычно говорим, что оно «повторяется» или что оно имеет «(алгебраическую) кратность $2$». Подумайте о том, что значит «повторить» собственное значение; при каких обстоятельствах мы перечисляем его дважды или больше? И когда мы это сделаем, сколько раз мы должны перечислять это?

Похоже, вы считаете, основываясь на размерности собственного пространства для собственного значения (или, что то же самое, на максимальном количестве линейно независимых собственных векторов, которое вы можете найти). n$, и, как следствие, мы всегда можем сформировать базис обобщенных собственных векторов. Есть особенно хороший класс таких оснований, называемых основаниями Джордана; это следующие лучшие вещи, которые мы можем найти после баз собственных векторов. Вместо того, чтобы диагонализовать матрицу, они превращают ее в нормальную форму Жордана — отличный утешительный приз, когда нам отказывают в диагональном представлении. Нормальные формы Жордана существуют для каждая матрица , в отличие от диагональных форм!

Алгебраические кратности также соответствуют показателям соответствующих множителей в характеристических полиномах. Фактически, некоторые определяют характеристический многочлен по этой характеристике, и его детерминантное представление становится теоремой.


В случае представленной матрицы $2 \times 2$ собственное пространство $\operatorname{ker} (M — I)$ равно просто $\operatorname{span}\{(1, 0)\}$. Однако, если мы вычислим $$\operatorname{ker} \left(\begin{bmatrix} 1 & 1 \\ 0 & 1 \end{bmatrix} — \begin{bmatrix} 1 & 0 \\ 0 & 1 \end{bmatrix}\right) ^2 = \operatorname{ker} \begin{bmatrix} 0 & 1 \\ 0 & 0 \end{bmatrix}^2 = \operatorname{ker} \begin{bmatrix} 0 & 0 \\ 0 & 0 \end{ bmatrix} = \Bbb{C}^2,$$ мы видим, что обобщенное собственное пространство $2$-мерно, а алгебраическая кратность равна $2$. n=0$ для некоторого $n$, существует некоторый базис $\mathcal{B}=\{b_1, …,b_n\}$ такой, что матрица в этом базисе имеет вид матрицы сдвига (т.е. $A b_i= b_{i+1}$ или $A b_i=0$ для любого $i$).

Существует еще одна теорема, утверждающая, что любая матрица является суммой нильпотентной матрицы и матрицы, которую можно диагонализовать (при условии, что ваши скаляры принадлежат алгебраически замкнутому полю, такому как $\mathbb{C}$). Следовательно, возьмем вашу общую $M=D+A,$, где $D$ диагонализируема, а $A$ нильпотентна. Затем приведенные выше наблюдения позволяют нам найти базис, такой, что $D$ является диагональным, и другой базис, такой, что $A$ является матрицей сдвига.

Однако магия обычной формы Джордана в том, что это можно делать одновременно. То есть существует единственный базис, относительно которого $D$ диагональна, а $A$ является матрицей сдвига. Таким образом, препятствием к геометрической кратности каждого собственного значения, соответствующего их алгебраической кратности, является именно эта матрица сдвига $A$. Если $A=0,$, то $D$ диагонализируема и, конечно, наоборот.

В вашем случае у вас есть что-то, что почти собственный вектор, а именно $(0,1),$, но вместо того, чтобы просто производить скаляр, кратный самому себе, он производит и честный собственный вектор , $(1,0)$. Они называются обобщенными собственными векторами.

Итак, резюмируем: с геометрической точки зрения у вас не так много реальных собственных векторов, как хотелось бы, потому что происходит некоторый сдвиг.

$\endgroup$

1

9кх = 0 $$ для некоторого натурального $k$. Для каждого собственного значения $\lambda$ вы получаете соответствующее подпространство, где $B-\lambda I$ действует как нильпотентная матрица. Его размерность равна алгебраической кратности $\lambda$ (поэтому алгебраическая кратность и обобщенная геометрическая кратность совпадают). Если алгебраическая кратность и геометрическая кратность $\lambda$ совпадают, то $B-\lambda I$ действует как нулевая матрица на этом подпространстве.

Если вы используете это, чтобы «диагонализировать» $B$ как можно лучше, вы получите так называемую Нормальная форма Жордана (с $\lambda$ по диагонали и $1$ по наддиагонали в подходящих местах).

В вашем случае матрица уже находится в жордановой нормальной форме, и вся плоскость (за вычетом начала координат) представляет собой обобщенные собственные векторы с собственным значением $1$.

$\endgroup$

$\begingroup$

Полезнее думать, что дефектные матрицы не имеют лишних собственных значений, а имеют меньше «правильных» собственных векторов. Любая матрица размера n x n имеет n собственных значений (включая кратность корней характеристического уравнения), но любые идентичные собственные значения (кратности m) могут иметь общий собственный вектор. Инвариантное подпространство оператора, связанного с этим собственным значением, по-прежнему имеет основу из m независимых векторов, но они не являются чистыми участками, как определяются собственные векторы. Вместо этого они образуют цепочку обобщенных собственных векторов, которые преобразуются от одного к другому с помощью нильпотентной части оператора, как это диктуется нормальной формой Жордана.

$\endgroup$

Твой ответ

Зарегистрируйтесь или войдите в систему

Зарегистрируйтесь с помощью Google

Зарегистрироваться через Facebook

Зарегистрируйтесь, используя электронную почту и пароль

Опубликовать как гость

Электронная почта

Требуется, но никогда не отображается

Опубликовать как гость

Электронная почта

Требуется, но не отображается

Нажимая «Опубликовать свой ответ», вы соглашаетесь с нашими условиями обслуживания, политикой конфиденциальности и политикой использования файлов cookie

.

Как узнать, что матрица неисправна? – КороткийИнформер

на Рекомендации

Emma Jonson

Содержание

Как узнать, что матрица неисправна?

В линейной алгебре дефектная матрица — это квадратная матрица, не имеющая полного базиса собственных векторов и поэтому не поддающаяся диагонализации. В частности, матрица размера n × n дефектна тогда и только тогда, когда она не имеет n линейно независимых собственных векторов.

Что такое исправная матрица?

Бездефектные матрицы — это именно те матрицы, которые имеют разложение по собственным значениям. Теорема. A ∈ IRm×m недефектна тогда и только тогда, когда она имеет собственное значение. разложение. A = XΛX−1 Ввиду этого другой член для недефектности диагонализируем.

Является ли сингулярная матрица дефектной?

Матрица A имеет 0 в качестве одного из собственных значений тогда и только тогда, когда она сингулярна. Определение дефектной матрицы: матрица A дефектна, если A имеет хотя бы одно собственное значение, геометрическое кратное которого. строго меньше своего алгебраического мульта.

При каких значениях A матрица дефектна?

Матрица A называется дефектной, если A имеет собственное значение λ кратности m>1, для которого соответствующее собственное пространство имеет базис менее чем из m векторов; то есть размерность собственного пространства, связанного с λ, меньше m.

Может ли дефектная матрица быть симметричной?

14. 3 Собственные векторы симметричных матриц. Вещественные симметричные матрицы (или, в более общем смысле, комплексные эрмитовы матрицы) всегда имеют действительные собственные значения и никогда не бывают дефектными. Однако после этого необходимо сделать собственные векторы в этом классе ортонормированными.

Обратимы ли дефектные матрицы?

Заметим, что неверно утверждать, что всякая обратимая матрица диагонализируема. А=[1101]. Определитель A равен 1, следовательно, A обратим. Поскольку геометрическая кратность строго меньше алгебраической, матрица A дефектна и не диагонализируема.

Можно ли любую матрицу диагонализовать?

Каждая матрица не диагонализируема. Возьмем, к примеру, ненулевые нильпотентные матрицы. Разложение Жордана говорит нам, насколько близка данная матрица к диагонализируемости.

Может ли симметричная матрица иметь повторяющиеся собственные значения?

(i) Все собственные значения симметричной матрицы действительны, а значит, и собственные векторы. Если симметричная матрица имеет какие-либо повторяющиеся собственные значения, все еще возможно определить полный набор взаимно ортогональных собственных векторов, но не каждый полный набор собственных векторов будет обладать свойством ортогональности.

Диагонализуемые матрицы обратимы?

Нет. Например, нулевая матрица диагонализируема, но необратима. Квадратная матрица обратима тогда и только тогда, когда ее ядро ​​равно 0, а элемент ядра — это то же самое, что и собственный вектор с собственным значением 0, поскольку он отображается в 0 раз сам, что равно 0,9.0005

Можно ли диагонализовать матрицу с 0 собственными значениями?

имеет 0 в качестве единственного собственного значения, но это не нулевая матрица, и поэтому ее нельзя диагонализовать. Ясно, что если N — нильпотентная матрица (т. е. Nk = 0 для некоторого k), то она диагонализируема тогда и только тогда, когда N = 0. Коэффициенты характеристического многочлена матрицы A являются многочленами от элементов матрицы A.

Существует ли такое понятие, как бракованная матрица?

Эрмитова матрица (или частный случай вещественной симметричной матрицы) или унитарная матрица никогда не бывают дефектными; в более общем случае нормальная матрица (которая включает эрмитову и унитарную как частные случаи) никогда не бывает дефектной.

Можно ли диагонализовать матрицу, если собственное значение дефектно?

Убедитесь, что нет дефектных собственных значений. Если какое-либо собственное значение дефектно, то матрица не может быть диагонализирована. В противном случае вы можете перейти к следующему шагу. Для каждого собственного значения найдите как можно больше линейно независимых собственных векторов (их количество равно геометрической кратности собственного значения).

Как в дефектной матрице образуется полный базис?

Полный базис формируется путем пополнения собственных векторов обобщенными собственными векторами, которые необходимы для решения дефектных систем обыкновенных дифференциальных уравнений и других задач. Дефектная матрица размера n × n всегда имеет менее n различных собственных значений, поскольку различные собственные значения всегда имеют линейно независимые собственные векторы.

Когда диагональная матрица является обратимой матрицей?

Другими словами, если диагонально, то существует обратимая матрица такая, что где — диагональная матрица, то есть матрица, недиагональные элементы которой равны нулю. Пример Определите матрицу и Обратное преобразование подобия дает в результате диагональную матрицу.

Отчет о клиническом случае: Хирургическое закрытие хронических дефектов мягких тканей с использованием трансплантата внеклеточного матрикса с наращенными тканевыми лоскутами

Введение

Реконструкция лоскутом является хорошо зарекомендовавшим себя подходом к закрытию хронических дефектов мягких тканей, однако относительно часто встречаются послеоперационные осложнения, такие как инфекция, расхождение швов и рецидивы. Долгосрочный успех закрытия лоскута дополнительно осложняется сопутствующими заболеваниями пациента, такими как ожирение, диабет и венозная недостаточность. Ретроспективный анализ 755 пролежней, излеченных 90 216 с помощью закрытия 90 217 лоскутов, показал, что общая частота осложнений составила 25% при 30-дневном наблюдении (1). Проспективное исследование 276 пролежней, закрытых продвижением лоскута, показало частоту осложнений 58%, где наиболее частыми осложнениями были расхождение швов раны (31,2%) и рецидив (28,6%) (2). Эти осложнения, связанные с закрытием лоскутом хронических дефектов мягких тканей, вероятно, связаны с плохим качеством подлежащих тканей, которые могут быть фиброзными и/или воспаленными, потенциалом мертвого пространства между продвигающимся лоскутом и подлежащими тканями и плохой васкуляризацией тканей в Генеральная.

Трансплантаты внеклеточного матрикса (ECM) представляют собой рассасывающиеся биокаркасы, обычно используемые в ряде пластических и реконструктивных процедур для восстановления мягких тканей. Эти технологии обеспечивают временный каркас для клеточной инфильтрации и образования капилляров, обеспечивая при этом защитное покрытие и укрепление дефекта до тех пор, пока биокаркас не абсорбируется регенерирующими мягкими тканями (3). Многие различные трансплантаты ECM клинически доступны и различаются по происхождению исходной ткани (например, человека, свиньи, крупного рогатого скота, лошади) и процессам, используемым для децеллюляризации ткани для удаления ядерного и клеточного материала при сохранении структуры и состава ткани. ЕСМ.

Матрикс овечьего преджелудка (OFM) представляет собой децеллюляризованный биокаркас ECM, выделенный из ткани овечьего преджелудка, который давно используется в ряде клинических применений, таких как лечение острых и хронических ран (4–9), пересадка кожи (10) и восстановление брюшной стенки (11, 12). Предыдущие исследования показали, что OFM выполняет множество биологических функций. Например, OFM оказывает противовоспалительное действие с модуляцией тканевых протеаз широкого спектра действия (13, 14), а также стимулирует миграцию, дифференцировку и инфильтрацию клеток (15, 16). Матрица способствует неоваскуляризации и заселена посредством клеточной инфильтрации и полностью ремоделируется в регенерирующие ткани (16). Понимание механизмов, лежащих в основе этих биологических эффектов, дает анализ структуры и состава OFM. На сегодняшний день в материале идентифицирован 151 различный матрисомальный белок, который включает широкий спектр коллагенов, белков адгезии и сигнальных молекул, таких как 12 факторов роста, включая, помимо прочего, фактор роста фибробластов 2 (FGF2), фактор роста тромбоцитов. (PDGF), эпидермальный фактор роста (EGF) и фактор роста соединительной ткани (CTGF) (15, 17). Структурные исследования показали, что целостность коллагеновых фибрилл и функциональные реакции сохраняются при OFM, что отражает сохранение нативной архитектуры ECM (18). Этот материал является высокопористым и способствует впитыванию жидкости и клеточной инфильтрации, а также обладает прочными механическими свойствами, подходящими для включения в устройства для индикации несущей нагрузки (19).).

Принимая во внимание регенеративные свойства трансплантатов ECM, было высказано предположение, что осложнения после реконструкции лоскутом хронических ран могут быть уменьшены за счет включения трансплантата ECM для стабилизации и наращивания хирургического лоскута и подлежащих тканей. В этой серии пилотных случаев представлены наши первоначальные результаты реализации этой стратегии.

Материалы и методы

Ретроспективные данные были собраны из протоколов операций, клинических фотографий и историй болезни. Всего n = 9 дефектов от 9 пациентов были включены в серию случаев (таблица 1). Все пациенты имели различные сопутствующие заболевания, осложняющие заживление, и должны были пройти плановую пластику лоскутом хронического дефекта. Все дефекты были хроническими и незаживающими в возрасте от 5 месяцев до 7 лет. Все дефекты препарировали путем резкой хирургической обработки или агрессивного иссечения хронических тканей. Трансплантат OFM ECM (Myriad™ Soft Tissue Matrix, Aroa Biosurgery Limited, Окленд, Новая Зеландия) регидратировали в стерильном физиологическом растворе (~ 5 мин), обрезали по размеру, а затем поместили в основание дефекта. Затем дефекты закрывались путем местного продвижения лоскута и наложения монокрилового шва 3-0. При необходимости использовались дренажи Джексона-Пратта (JP) и инцизионная терапия ран с отрицательным давлением (NPWT) (таблица 1). Дефекты наблюдали до 3–6 месяцев (таблица 1) на предмет расхождения швов, инфицирования или рецидива.

Таблица 1 . Участники.

Результаты и примеры клинических случаев

Мы предположили, что одновременное размещение трансплантата ВКМ в нижней части во время пластики хронических ран лоскутом может уменьшить хирургические осложнения за счет уменьшения воспаления проксимальных тканей и стабилизации лоскута. Используя эту стратегию, было достигнуто неосложненное заживление у n = 7 из n = 9 участников исследования с дефектами (таблица 1). Послеоперационное осложнение в виде расхождения швов возникло у n = 2 дефекта. Тем не менее, оба эти дефекта зажили вторично без дополнительного хирургического вмешательства. Все зажившие дефекты демонстрировали хорошую косметику, сравнимую с соседними тканями. Оценка заживших дефектов показала отличную функциональность и подвижность, и все пациенты были удовлетворены своими соответствующими результатами.

Случай 1

53-летний мужчина с ожирением, которому 10 лет назад была проведена однопортовая лапароскопическая установка желудочного бандажа. Полоска прорвалась через кожу и была удалена хирургическим путем, но закрытие осложнилось вторичным расхождением швов. На момент вмешательства у пациента в течение 11 мес имелась незаживающая рана живота, более поздняя ревизионная операция (5 мес назад) также оказалась неудачной с вскрытием операционного поля. Ранее проводилось лечение дефекта с помощью альгинатных повязок , йодного кадексмера и марли с физиологическим раствором. Дефект вызывал умеренную боль, но был чистым, без признаков инфекции и присутствовала гипертрофическая грануляционная ткань (рис. 1А). Хирургический разрез был сделан по краям дефекта, через подкожно-жировую клетчатку и до фасции (рис. 1Б). Трансплантат ECM (Myriad™, «Толстый») был обрезан, чтобы соответствовать дефекту, и помещен на фасцию (рис. 1C). Из-за глубины дефекта и возможного образования мертвого пространства второй и третий слои трансплантата ВКМ были подготовлены и размещены в дефекте послойно. Затем дефект был закрыт продвижением лоскута подкожным непрерывным швом с установленным дренажем JP и начата инцизионная NPWT. Через семь дней кожные ткани продемонстрировали хорошее прилегание, а в тканях не было воспаления (рис. 1D), NPWT и дренаж были удалены, и пациент был выписан с инструкцией носить абдоминальный бандаж и ограничение подъема тяжестей/физической активности в течение 6 недель. Через 6 месяцев узел оставался закрытым без расхождения швов или других осложнений.

Рисунок 1 . Выдвижение лоскута и стабилизация ECM расхождения брюшной полости. Случай 1 — (A) Эрозия тканей брюшной полости в результате бандажирования желудка и неудачной предыдущей хирургической реконструкции дефекта возрастом около 11 месяцев. (Б) Широкое иссечение дефекта до подлежащей фасции и жировой клетчатки. (C) Размещение трансплантата ECM в основание дефекта перед продвижением и закрытием лоскута. (D) Через семь дней после операции. Остался зажившим через 6 месяцев.

Случай 2

Семидесятитрехлетняя женщина, неспособная к передвижению вследствие болезни Паркинсона, волчанки и ревматоидного артрита. Травма крестцового давления присутствовала в течение 7 лет и ранее лечилась медицинским медом и ферментативной обработкой раны. За 1 месяц до этого пациенту была проведена реконструктивная операция с иссечением дефекта и укрытием лоскутом. Впоследствии лоскут раскрылся, и хотя признаков инфекции не было, дефект был хронически воспален (рис. 2А). Дефект иссечен боковыми краями до копчика с частичной остеотомией (рис. 2Б). Трансплантат ECM (Myriad™, «Толстый») был обрезан, чтобы соответствовать дефекту и в основании дефекта и покрыть костный выступ копчика (рис. 2C). Иссечение было закрытым через продвижение лоскута с швом, используемым для закрытия основной линии разреза (рис. 2D). Были установлены дренажи JP и начата инцизионная NPWT. Развилось раскрытие узла, которое впоследствии зажило посредством вторичного натяжения через 3 месяца. Через 6 мес дефект остался зажившим.

Рисунок 2 . Хирургическая реконструкция резистентной травмы крестцового сдавления. Случай 2 — (A) Пролежневая травма одиннадцатимесячной давности, которая ранее не удалась при реконструкции лоскутом 1 месяц назад; вторичная процедура, выполненная с использованием трансплантата ECM для стабилизации лоскута. (Б) Иссечение язвы с частичной резекцией. (C) Размещение трансплантата ECM и (D) закрытие лоскута.

Случай 3

Женщина, 26 лет, с 13-месячным незаживающим полнослойным дефектом после операции открытой репозиции и внутренней фиксации левой лодыжки (рис. 3А). Ортопедические изделия остались на месте, посев раны дал отрицательный результат. Ранее дефект лечили с помощью альгинатных повязок. Было проведено хирургическое иссечение проксимальных хронических тканей (рис. 3В). Обрезанный трансплантат ECM (Myriad™ «Thin») помещали в контакт с тканями ложа дефекта и давали регидратироваться in situ через абсорбцию компонентов крови (рис. 3B). Дефект был закрыт продвижением лоскута (рис. 3C) и выполнен инцизионной NPWT в течение семи дней. Дефект показал признаки инфекции и вторичного расхождения краев через 1 неделю после операции (рис. 3D). Дополнительный материал трансплантата ECM был упакован в полость раскрытия (рис. 3D) и покрыт вторичной повязкой из пены генцианвиолета/метиленового синего. В течение следующих недель дефект продемонстрировал заметное улучшение с развитием здоровой грануляционной ткани (рис. 3Е). Таким образом, для заживления дефекта было использовано наложение дополнительного трансплантата ECM, гидролизованных коллагеновых повязок, покрытых неприлипающим слоем повязки, и дальнейшее использование вторичных повязок с генцианвиолетовым/метиленовым синим через вторичное намерение. Успешное закрытие дефекта вторичным натяжением было достигнуто через 8 недель после первоначальной лоскутной операции (рис. 3F).

Рисунок 3 . Реконструкция незаживающего хирургического дефекта нижней конечности. Случай 3 — (A) Тринадцатимесячный незаживающий дефект после операции открытой репозиции и внутренней фиксации левой лодыжки. (Б) Хирургическое иссечение проксимальных хронических тканей и размещение трансплантата ВКМ в дефект. (C) Выдвижение и закрытие лоскута. (D) Дефект показал признаки инфекции и вторичное расхождение швов через 1 неделю после операции и был обработан вторым трансплантатом ECM. (E) Через 6 недель дефект продемонстрировал заметное улучшение с развитием здоровой грануляционной ткани. (F) Успешное закрытие дефекта вторичным натяжением достигнуто через 8 недель после первичной пластики лоскутом без дополнительного хирургического вмешательства.

Обсуждение

Мы предположили, что осложнения после реконструкции лоскутом хронических дефектов могут быть уменьшены за счет дополнительного использования трансплантата ECM для стабилизации и наращивания хирургического лоскута и подлежащих тканей. В этой серии пилотных случаев изучалась одноэтапная процедура реконструкции лоскута, дополненная размещением трансплантата ECM. Одновременное использование трансплантатов ECM для закрытия лоскута почти не описано в литературе. Было показано, что использование трансплантата внеклеточного матрикса, полученного из амниона, улучшает выживаемость случайного лоскута в доклинической мышиной модели (20), однако эта работа в основном была сосредоточена на комбинированном использовании трансплантата амниона с добавлением мезенхимальных стволовых клеток и не была переведена на исследования человека. Имеются различные клинические отчеты об использовании трансплантатов ECM для спасения поврежденных или несостоятельных лоскутов (21, 22), но эти отчеты описывают реактивное использование трансплантатов ECM во втором вмешательстве, а не преднамеренную первоначальную стратегию. В пародонтальной хирургии трансплантаты ECM обычно используются в сочетании с коронально расположенными лоскутами для лечения рецессии десны, обеспечивая покрытие корня и увеличение толщины десны (23, 24).

Насколько нам известно, это первая опубликованная работа по использованию трансплантатов ECM для увеличения реконструкции лоскута. Трансплантат ВКМ, использованный в настоящем исследовании, получен из ткани преджелудка овцы (овцы), в частности подслизистой основы, слоя ВКМ, который проходит через ткань преджелудка (15). После выделения подслизистая оболочка подвергается процессу децеллюляризации для удаления овечьих клеток и нуклеиновых кислот, оставляя слой интактного ВКМ. Децеллюляризация использует комбинацию детергентов, хелатирующих агентов и солей, чтобы сначала лизировать клетки овец, а затем растворить клеточные мембраны и нуклеиновые кислоты (15). Трансплантаты изготавливаются из отдельных слоев ВКМ и представлены в виде трех- (~1,0 мм) или пятислойного трансплантата (~1,5 мм) размером до 200 см 9 .0316 2 . В то время как усовершенствованные биокаркасы ECM, как правило, были труднодоступны из-за стоимости, цена трансплантата OFM ECM (~ 250–2500 долларов США) позволяет использовать эту технологию в реконструктивных процедурах по-новому.

Мы предположили, что трансплантат ECM может помочь уменьшить осложнения лоскута посредством комбинации биологических и физически опосредованных механизмов. Например, эффекты OFM, модулирующие протеазу (13), могут исправить основную несбалансированную среду при хронических дефектах мягких тканей с высоким уровнем воспаления тканей и протеолитической активностью. Стимулирование неоваскуляризации материалом ECM (16) может способствовать установлению кровоснабжения как из лоскута, так и из подлежащих тканей дефекта. Увеличение местного кровоснабжения дефекта за счет васкуляризации также сводит к минимуму риск инфекции через усиливают перфузию защитных элементов иммунной системы и/или системно вводимых антибиотиков (25). Действительно, правильная подготовка участка жизненно важна для успешного закрытия хронических дефектов, а положительные посевные культуры после санации являются доказанным предиктором неудачного закрытия лоскута (26), поскольку наша процедура включала агрессивную санацию и удаление хронически дефектных тканей. Кроме того, размещение материала трансплантата ВКМ в ложе дефекта может обеспечить окклюзию мертвого пространства между лоскутом и подлежащими мягкими тканями.

Результаты текущей серии пилотных случаев были обнадеживающими, но следует отметить ограниченный размер выборки. Кроме того, пять участников получили инцизионную NPWT, вклад которой не может быть полностью оценен при таком ограниченном размере выборки. Хотя известно, что инцизионная NPWT уменьшает инфекции в области хирургического вмешательства, было показано, что риск несостоятельности первичных хирургических швов эквивалентен стандартным раневым повязкам (27).

Заключительные замечания

Реконструкция лоскутом является эффективным и распространенным методом восстановления хронических дефектов мягких тканей. В настоящей серии случаев пилотное использование трансплантата ECM для увеличения реконструкции лоскутом хронических дефектов мягких тканей. Результаты этих первоначальных случаев могут потребовать будущих контролируемых исследований для оценки этой техники по сравнению с немодифицированным закрытием лоскута.

Заявление о доступности данных

Первоначальные материалы, представленные в исследовании, включены в статью/дополнительные материалы. Дальнейшие запросы можно направлять соответствующему автору/ам.

Заявление об этике

Все пациенты предоставили письменное информированное согласие на использование их изображений и данных в целях исследования и публикации.

Вклад автора

Доктор медицинских наук участвовал в разработке исследования, клиническом ведении, сборе данных, анализе данных и подготовке рукописи. KB и AW внесли свой вклад в ведение дел и сбор данных. К. Х., К. Д. и Д. Г. внесли свой вклад в клиническое ведение. Все авторы внесли свой вклад в доработку рукописи, прочитали и одобрили представленную версию.

Финансирование

Доктор медицинских наук получил образовательный грант на поездку от Aroa Biosurgery Limited.

Конфликт интересов

Доктор медицинских наук консультирует Aroa Biosurgery Limited (Окленд, Новая Зеландия). Матрица мягких тканей Myriad™ была предоставлена ​​компанией Aroa Biosurgery Limited.

Остальные авторы заявляют, что исследование проводилось в отсутствие каких-либо коммерческих или финансовых отношений, которые могли бы быть истолкованы как потенциальный конфликт интересов.

Благодарности

Авторы выражают благодарность Барнаби Мэю и Кристоферу Миллеру (Aroa Biosurgery Limited) за помощь в подготовке рукописи.

Ссылки

1. Tran BNN, Chen AD, Kamali P, Singhal D, Lee BT, Fukudome EY. Национальные периоперационные результаты закрытия лоскутов при пролежнях с 2005 по 2015 год с использованием Национальной программы повышения качества хирургии Американского колледжа хирургов. Арка Пласт Сург . (2018) 45:418. doi: 10.5999/aps.2018.00262

Полный текст перекрестной ссылки | Google Scholar

2. Bamba R, Madden JJ, Hoffman AN, Kim JS, Thayer WP, Nanney LB, et al. Реконструкция лоскутом при пролежнях: анализ результатов. Plastic Reconstr Surg Glob Open. (2017) 5:e1187. doi: 10.1097/GOX.0000000000001187

PubMed Abstract | Полный текст перекрестной ссылки | Google Scholar

3. С. Бадылак. Внеклеточный матрикс как каркас для реконструкции тканей. Semin Cell Dev Biol . (2002) 13:377–460. дои: 10.1016/S1084952102000940

Полнотекстовая перекрестная ссылка | Google Scholar

4. Феррерас Д.Т., Крейг С., Малкомб Р. Использование повязки из овечьего коллагена с интактным внеклеточным матриксом для сокращения времени закрытия ран и сокращения расходов в амбулаторном раневом центре госпиталя ветеранов войны США. Surg Technol Int. (2017) 30:61–9.

PubMed Abstract

5. Бон Г.А., Гасс К. Результаты лечения язвы голени новой повязкой из внеклеточного матрикса из овечьего коллагена: серия ретроспективных случаев. Уход за ранами Adv Skin . (2014) 27:448–454. doi: 10.1097/01.ASW.0000453728.12032.6f

PubMed Abstract | Полный текст перекрестной ссылки | Google Scholar

6. Лиден Б.А., Май до н.э. Клинические результаты после использования матрикса овечьего преджелудка (эндоформного кожного шаблона) для лечения хронических ран. Средство для ухода за ранами Adv Skin . (2013) 26:164–7. doi: 10.1097/01.ASW.0000428862.34294. d4

PubMed Abstract | Полный текст перекрестной ссылки | Google Scholar

7. Simcock J, Than M, Ward B, May B. Лечение изъязвленного липоидного некробиоза матриксом овечьего преджелудка. J Уход за ранами . (2013) 22:383–4. doi: 10.12968/jowc.2013.22.7.383

PubMed Abstract | Полный текст перекрестной ссылки | Google Scholar

8. González: Применение повязки с коллагеновым внеклеточным матриксом для лечения рецидивирующей венозной язвы у 52-летнего пациента. J Уход за раной для ухода за стомой . (2016) 43:310–2. doi: 10.1097/WON.0000000000000231

CrossRef Full Text | Google Scholar

9. Э. Дж. Луллов. Использование внеклеточного матрикса на основе овечьего коллагена и антибактериальных губчатых повязок на основе генцианвиолета/метиленового синего для улучшения клинических результатов при ранениях нижних конечностей: ретроспективное когортное исследование. Ранения . (2017) 29:107–114.

10. Симкок Дж., Май до н. э. Матрица преджелудка овцы как субстрат для одноэтапной реконструкции расщепленного трансплантата. Эпластика . (2013) 13:e58.

Реферат PubMed | Google Scholar

11. Ferzoco SJ. Ранний опыт применения усиленных биокаркасов при пластике паховой грыжи: серия случаев. Int J Surg Open. (2018) 12:9–11. doi: 10.1016/j.ijso.2018.06.001

Полный текст CrossRef | Академия Google

12. Сойер М.А. Новый армированный полимером биокаркас из овечьего молока для лечения грыж пищеводного отверстия диафрагмы. JSLS. (2018) 22:e2018.00057. doi: 10.4293/JSLS.2018.00057

PubMed Abstract | Полный текст перекрестной ссылки | Google Scholar

13. Негрон Л., Лунь С., Май до н.э. Биоматериал матрикса преджелудка овец является ингибитором матриксных металлопротеиназ и эластазы нейтрофилов широкого спектра действия. Внутренняя рана J . (2014) 11:392–7. doi: 10.1111/j.1742-481X.2012.01106.x

PubMed Abstract | Полный текст перекрестной ссылки | Академия Google

14. Street M, Thambyah A, Dray M, Amirapu S, Tuari D, Callon KE, et al. Аугментация матричным каркасом из овечьей преджелудки улучшает гистологические результаты восстановления ротаторной манжеты плеча на модели крысы. J Orthop Surg Res . (2015) 10:165. doi: 10.1186/s13018-015-0303-8

PubMed Abstract | Полный текст перекрестной ссылки | Google Scholar

15. Лун С., Ирвин С.М., Джонсон К.Д., Фишер Н.Дж., Флоден Э.В., Негрон Л. и др. Биоматериал функционального внеклеточного матрикса, полученный из овечьего преджелудка. Биоматериалы . (2010) 31:4517–29. doi: 10.1016/j.biomaterials.2010.02.025

PubMed Abstract | Полный текст перекрестной ссылки | Google Scholar

16. Ирвин С.М., Кайзер Дж., Тодд Э.М., Лун С., Флоден Э.В., Негрон Л. и др. Количественная оценка ангиогенеза in vitro и in vivo, стимулированного биоматериалом матрикса овечьего преджелудка. Биоматериалы . (2011) 32:6351–61. doi: 10.1016/j.biomaterials.2011.05.040

PubMed Abstract | Полный текст перекрестной ссылки | Академия Google

17. Демпси С.Г., Миллер С.Х., Хилл Р.С., Хансен К.С., Мэй до н.э. функциональные идеи из протеомного инвентаря матрикса овечьего преджелудка. J Протеом Res . (2019) 18:1657–68. doi: 10.1021/acs.jproteome.8b00908

PubMed Abstract | Полный текст перекрестной ссылки | Google Scholar

18. Sizeland KH, Wells HC, Kelly SJ, Nesdale KE, May BC, Dempsey SG, et al. Реакция коллагеновых фибрилл на растяжение каркасов из овечьего преджелудка для тканевой инженерии. ACS Biomater Sci Eng . (2017) 3:2550–8. doi: 10.1021/acsbimaterials.7b00588

Полный текст CrossRef | Google Scholar

19. Флоден Э.В., Малак С.Ф., Бэзил-Джонс М.М., Негрон Л., Фишер Дж.Н., Лун С. и др. Биофизическая характеристика биоматериалов внеклеточного матрикса преджелудка овец. Журнал исследований биомедицинских материалов, часть B: Прикладные биоматериалы . (2011) 96:67–75. doi: 10.1002/jbm.b.31740

PubMed Abstract | Полный текст перекрестной ссылки | Google Scholar

20. Chehelcheraghi F, Eimani H, Homayoonsadraie S, Torkaman G, Amini A, Majd HA, et al. Влияние бесклеточного матрикса амниотической мембраны и мезенхимальных стволовых клеток костного мозга на улучшение выживаемости случайного кожного лоскута у крыс. Медицинский журнал Красного Полумесяца Ирана J. (2016) 18:e25588. doi: 10.5812/ircmj.25588

PubMed Abstract | Полный текст перекрестной ссылки | Google Scholar

21. Шанти Р. М., Смарт Р. Дж., Мерам А., Ким Д. Внеклеточный матрикс мочевого пузыря свиньи для спасения кожного лоскута малоберцовой кости: техническое примечание и описание случая. Реконструкция черепно-челюстной травмы . (2017) 10:318–22. doi: 10.1055/s-0036-1593473

PubMed Abstract | Полный текст перекрестной ссылки | Академия Google

22. Kruper GJ, VandeGriend ZP, Lin H-S, Zuliani GF. Спасение неудачных местных и регионарных лоскутов внеклеточным матриксом мочевого пузыря свиньи способствовало регенерации тканей. Представитель Отоларингол. (2013) 2013:3. doi: 10.1155/2013/3

PubMed Abstract | Полный текст перекрестной ссылки | Google Scholar

23. Guan W, Liao H, Guo L, Wang C, Cao Z. Закрытие корня с использованием коронально выдвинутого лоскута с бесклеточным дермальным матриксом или без него: метаанализ. J Пародонтальный имплантат Sci . (2016) 46:22–34. doi: 10.5051/jpis.2016.46.1.22

CrossRef Полный текст

24. de Queiroz Côrtes A, Sallum AW, Casati MZ, Nociti FH Jr, Sallum EA. Двухлетнее проспективное исследование коронарно расположенного лоскута с трансплантатом бесклеточного дермального матрикса или без него. J Clin Пародонтол . (2006) 33:683–9. doi: 10.1111/j.1600-051X.2006.00969.x

Полный текст CrossRef | Google Scholar

25. Suh HP, Hong JP. Роль реконструктивной микрохирургии в лечении хронических ран нижних конечностей. Int Wound J. (2019) 16:951–9. doi: 10.1111/iwj.13127

PubMed Abstract | Полный текст перекрестной ссылки | Google Scholar

26. Шак Дж., Нолан Дж., Канури А., Эванс К.К., Аттингер К.Э. Влияние положительных посевов после санации на локальную реконструкцию мышечного лоскута нижней конечности. Plastic Reconstr Surg. (2015) 136:9–10. doi: 10.1097/01.prs.0000472283.01460.a1

PubMed Abstract | Полный текст перекрестной ссылки | Google Scholar

27. Norman G, Goh EL, Dumville JC, Shi C, Liu Z, Chiverton L, et al. Раневая терапия отрицательным давлением для заживления хирургических ран первичным закрытием. Cochrane Database Syst Rev. (2020) 6:CD009261. doi: 10.1002/14651858.CD009261.pub5

PubMed Abstract | Полный текст перекрестной ссылки | Google Scholar

Матрица ОК – постоянное совершенствование производства

Матрица обеспечения качества (матрица ОК)

Матрица ОК – это таблица, в которой описывается, как дефекты появляются в продуктах в текущих условиях, в процессе, в котором нужно убедиться качественный. Эта матрица позволяет менеджерам сразу увидеть, какие процессы порождают дефекты, оборудование и методы каких процессов неисправны и как эти два процесса связаны между собой.

Подготовка матрицы контроля качества

  • Проверка стандартов для рассматриваемого продукта и выявление всех характеристик качества и типов дефектов, которые, как считается, препятствуют выполнению этих стандартов.
  • Нарисуйте блок-схему для обеспечения качества во всех процессах, охватывающих каждый процесс (от основных процессов до вспомогательных процессов),
  • Изучите и подтвердите статус возникновения дефекта и отдельных процессов (от основных процессов до вспомогательных процессов).
  • Выделите все характеристики качества и отобразите их виды дефектов.
  • Классифицируйте характеристики качества в соответствии со степенью важности их видов дефектов (ранжирование характеристик), указывая прошлые дефектные результаты, которые вас интересуют.
  • Повторите эту процедуру до мельчайших единиц процесса, чтобы была видна взаимосвязь с видами дефектов.

Ключевые моменты

  • Попросите менеджера, ответственного за отдел обеспечения качества, изучить классификации по степени важности (характеристический рейтинг), чтобы выполнить классификацию с точки зрения типов дефектов.
  • Все ответственные руководители принимают участие в анализе относительности, анализируя отдельные процессы и виды дефектов.

Примеры основных и вспомогательных процессов

Пример: Продукт А (процесс сушки)

Идентификация элементов качества продукта (требуемые характеристики качества) — Определение стандартов на продукт

Примеры стандартов на продукт
  • Материалы — Ингредиенты, структура и т. д.
  • Точность формы — Округлость, параллельность, перпендикулярность и т. д.
  • Точность размеров. Диаметр, наружный диаметр, длина, толщина и т. д.
  • Точность внешнего вида. Посторонние включения, отклонения в цвете, царапины, трещины и т. д.
  • Химическое качество. сопротивление и т. д.
  • Электрические качества — Изоляция, электрическая проводимость, проводимость и т. д.

Стандарты на продукцию — Точность качества, определяемая конструкцией. Эти стандарты продукта всегда определяются такими факторами, как допуск и область применения.

Качество дизайна — Превосходные продукты не могут быть созданы просто на основе функциональных критериев, таких как «легкость переноски» или «компактная, удобная для переноски форма». Скорее, элементы качества дизайна, такие как форма, размер и вес, которые являются характеристиками качества, соответствующими этим элементам, должны быть определены до того, как можно будет приступить к конкретным усилиям по проектированию.

Подготовка матрицы ОК (образец)

Матрица поддержания качества (УК)

Подготовка матрицы УК

  1. Отобразите в матричной таблице QM взаимосвязь между характеристиками качества и элементами обследования из таблицы анализа PM (включая элементы, которые не вызвали проблем в оценке обследования), направленными на хронические дефекты. Объекты обследования рассматриваются как факторы хронических дефектов.
    • Осмотрите основные составляющие системы таблицы анализа PM (содействующие факторы) в качестве репрезентативной выборки.
  2. На этапах анализа PM изучите элементы управления и их методы проверки для элементов качества, для которых меры по предотвращению отклонений были предприняты на шаге 4, а также другие характеристики качества и наметьте их взаимосвязь с характеристиками качества в матрице QM стол.
  3. Оцените уровень воздействия элементов управления на качество и ранжируйте элементы управления следующим образом: ◎: сильное влияние на качество, ○: среднее влияние на качество или △: низкое влияние на качество.
  4. Определите элементы управления метода проверки и порог управления, цикл проверки (ссылаясь на уровень воздействия на элементы качества) и инспекторов.
  5. Добавьте условия управления в документы стандартов автономного обслуживания оборудования и контрольные листы автономного обслуживания.
  6. Добавьте их в чертежи процесса управления.

Стандартизация элементов контроля технического обслуживания

Внедрение шага 6 автономного технического обслуживания

Обычный подход к нарушениям заключается в том, что, как только становится ясно, что отклонения велики и существуют дефекты, тщательно выявляются лежащие в их основе факторы, чтобы принять соответствующие меры. можно сформулировать. Однако если дефекты уже начали появляться, значит, вы слишком долго ждали. При таком подходе постфактум к управлению, даже если достигается предотвращение повторения, невозможно свести дефекты к нулю. При поддержании качества, развертываемом на шаге 6, взаимосвязь между оборудованием и качеством, установленная на шаге 5, дополнительно исследуется. Точность всех связанных секций, включая оборудование, детали, приспособления и инструменты, измеряется до возникновения дефектов, чтобы убедиться, что они находятся в требуемом состоянии, чтобы можно было исправить любое отклонение от требуемого состояния.

Чтобы реализовать этот подход к обеспечению качества, проведите организацию и очистку элементов контроля, связанных с качеством, используя следующую процедуру.

Процедура

  1. Понимание функций рассматриваемых продуктов или деталей.
  2. Подтверждение стандартов качества и качественных характеристик продукции и готовой продукции с помощью таблицы процессов контроля качества.
  3. Организуйте возникающие дефекты в «список возникающих дефектов» с указанием типов дефектов, а также времени, места и способа их возникновения для каждого процесса.
  4. Уточнить конструкцию, условия обработки, методы работы и способы настройки оборудования, в котором возникают дефекты.
  5. Подготовьте матрицу QM, указывающую диапазон точности, требуемый для элементов, в которых меры против дефектов качества были выполнены в разделах, влияющих на качество.
  6. Добавьте элементы контроля, отображаемые в матрице QM, в контрольный список элементов планового контроля, используемых в настоящее время, и регулярно проверяйте, поддерживается ли постоянно требуемая точность.

Хондрогенез, индуцированный аутологичным матриксом, эффективен при очаговых хондральных дефектах коленного сустава

Abstract

Очаговые хондральные дефекты коленного сустава встречаются часто, и их лечение представляет собой сложную задачу. В этом исследовании изучалась эффективность и безопасность индуцированного аутологичным матриксом хондрогенеза (AMIC) при фокальных хондральных дефектах коленного сустава. Был проведен систематический обзор и метаанализ (согласно заявлению PRISMA 2020 г.) для изучения эффективности AMIC в улучшении симптомов и для сравнения AMIC с микропереломами (MFx). В январе 2022 г. осуществлялся доступ к следующим базам данных: Pubmed, Web of Science, Google Scholar, Embase. Для поиска не использовалось ограничение по времени. Были доступны все клинические испытания, посвященные AMIC и/или сравнивающие AMIC с MFx при фокальных хондральных дефектах коленного сустава. Учитывались только исследования, опубликованные в рецензируемых журналах. Исследования, в которых изучались дефекты в других местах, а не в колене, не соответствовали требованиям, а те, в которых сообщались данные, представляют собой смешанные места. Исследования, в которых сообщались данные о ревизионных настройках, а также исследования эффективности при поцелуях или нескольких местах, не подходили. Для непрерывных и бинарных данных использовали меру эффекта разности средних (MD) и отношения нечетностей (OR). Были получены данные 18 исследований (548 пациентов) со средним периодом наблюдения 39 пациентов.0,9 ± 26,5 месяцев. Средний размер дефекта составил 3,2 ± 1,0 см 2 . Визуальная аналоговая шкала (ВАШ) уменьшилась на - 3,9/10 (95% доверительный интервал (ДИ) от - 4,0874 до -3,7126), шкала активности Тегнера увеличилась на  + 0,8/10 (95% ДИ от 0,6595 до 0,9405). Система оценки коленного сустава Lysholm увеличилась на  + 28,9/100 (95% ДИ от 26,8716 до 29,1284), как и Международный комитет по документации коленного сустава (IKDC)  + 33,6/100 (95% ДИ от 32,5800 до 34,6200). При последнем осмотре ни у одного пациента не было выявлено признаков гипертрофии. 4,3% (9из 210) пациентов подверглись ревизионным вмешательствам. Частота неудач составила 3,8% (9 из 236). По сравнению с MFx, AMIC продемонстрировал более низкую оценку по ВАШ (РС: - 1,01; 95% ДИ от - 1,97 до 0,05), большую IKDC (РС: 11,80; 95% ДИ от 6,65 до 16,94) и более низкую частоту ревизий (ОШ: 0,16; 95). % ДИ от 0,06 до 0,44). AMIC эффективен при очаговых хондральных дефектах коленного сустава. Кроме того, AMIC свидетельствовал о большем IKDC, наряду с более низким значением VAS и частотой ревизий по сравнению с MFx.

Введение

Очаговые хрящевые дефекты коленного сустава часто встречаются 1,2 . Дефекты хряща отрицательно сказываются на занятиях спортом и на качестве жизни пострадавших пациентов 3 . Если не лечить, хондральные дефекты имеют ограниченные шансы на заживление, и может возникнуть хроническая боль 4,5,6 . Лечение хондральных дефектов является сложной задачей с непредсказуемыми результатами 7,8 . Для симптоматических дефектов размером менее 2 см 2 предложены микропереломы (MFx) 9,10,11,12 . MFx представляет собой простую процедуру стимуляции костного мозга, которую можно проводить полностью артроскопически 13 . Во время MFx хрящ очищается до его жизнеспособной границы, и выполняются микропереломы, чтобы способствовать миграции клеток из субхондральной кости 14,15 . Костный мозг является основным гемопоэтическим и лимфоидным органом, нишей для поддержки самообновления и дифференцировки гемопоэтических стволовых клеток (ГСК), мультипотентных клеток-предшественников (MPP) и коммитированных клеток-предшественников для продукции клеток крови 16,17,18 . Считается, что субхондральные клетки костного мозга улучшают восстановление хряща 19,20,21 . Однако при более крупных дефектах сгусток крови, образующийся после MFx, не обладает достаточной механической устойчивостью, чтобы оставаться на месте 22 . Чтобы преодолеть это ограничение, в 2005 г. Behrens et al. 23 впервые описал усовершенствованную технику микропереломов, которая превратилась в аутологичный матрикс-индуцированный хондрогенез (AMIC). При AMIC резорбируемая мембрана используется для стабилизации сгустка и поддержания его стабильности в полости сустава 24,25 . В отличие от других хрящевых процедур, AMIC не требует забора какой-либо аутологичной ткани и выполняется за один сеанс операции 26,27 . Благодаря этим особенностям AMIC представляет особый интерес как для пациентов, так и для хирургов 28 .

Опубликовано несколько клинических исследований по оценке эффективности и безопасности AMIC при фокальных хондральных дефектах коленного сустава 24,25,29,30,31,32,33,34,35,36,37 . Однако за последние несколько лет было опубликовано несколько исследований, которые еще не были включены в предыдущий обзор 9.0316 22,27,38,39,40,41,42 . Поэтому был проведен систематический обзор и метаанализ. Основная цель настоящего исследования заключалась в изучении эффективности и безопасности AMIC при очаговых хондральных дефектах коленного сустава. Вторичной целью было выяснить, способствует ли AMIC при очаговых хондральных дефектах коленного сустава лучшему результату, чем MFx. Мы предположили, что AMIC, выполняемая в коленном суставе, может быть эффективной и безопасной для лечения симптоматических дефектов хряща.

Методы

Критерии приемлемости

Доступ ко всем клиническим испытаниям, посвященным AMIC и/или сравнивающим AMIC с MFx при фокальных хондральных дефектах коленного сустава. Учитывались только исследования, опубликованные в рецензируемых журналах. Согласно языковым возможностям автора, к участию в конкурсе допускаются статьи на английском, немецком, итальянском, французском и испанском языках. Учитывались только исследования с уровнем доказательности от I до IV согласно Оксфордскому центру доказательной медицины 43 . Рецензии, мнения, письма, редакционные статьи не учитывались. Исследования, в которых изучались дефекты в других местах, а не в колене, не соответствовали требованиям, как и те, в которых сообщались данные из смешанных мест. Исследования, в которых сообщались данные о ревизионных установках, а также исследования эффективности этих техник при повреждениях при поцелуях или в нескольких местах, не подходили. Животные, in vitro, биомеханические, вычислительные и трупные исследования не подходили. Отсутствующие количественные данные по интересующим результатам служили основанием для исключения из исследования.

Стратегия поиска

Это исследование было проведено в соответствии с Предпочтительными элементами отчетности для систематических обзоров и метаанализов: заявлением PRISMA 2020 44 . Предварительно указан алгоритм PICOT:

  • P (Проблема): дефект коленного хряща;

  • I (Вмешательство): AMIC;

  • C (Сравнение): MFx;

  • O (Результаты): PROM, скорость гипертрофии, неудачи и повторная операция.

  • T (время): минимум 12 месяцев наблюдения.

В январе 2022 года был осуществлен доступ к следующим базам данных: Pubmed, Web of Science, Google Scholar, Embase. Ограничение по времени на поиск не устанавливалось. Для выполнения поиска в каждой базе данных использовалась следующая матрица ключевых слов: ( knee ) AND ( chondral defects OR chondropathy OR cartilage defects ) AND ( Autologous Matrix-Induced Chondrogenesis OR AMIC OR surgery AND microfractures ) AND ( pain OR symptoms ИЛИ исход И показатели исхода, сообщенные пациентом ИЛИ PROM) ИЛИ ( осложнения И пересмотр И гипертрофия И неудача ). Никаких дополнительных фильтров при поиске по базам данных не использовалось.

Отбор и сбор данных

Два автора (Ф. М. и Х. С.) независимо друг от друга выполнили поиск в базе данных. Все получившиеся заголовки проверялись вручную и, если они подходили, открывался доступ к реферату. Доступны полные тексты тезисов, соответствующих теме. Если полный текст был недоступен или недоступен, статья не рассматривалась для включения. Для включения также была выполнена перекрестная ссылка на библиографию полнотекстовых статей. Разногласия обсуждались и взаимно решались авторами. В случае дальнейших разногласий окончательное решение принимал третий старший автор (Н. М.).

Элементы данных

Два автора (Ф. М. и Х. С.) независимо выполнили извлечение данных. Были извлечены следующие исходные данные: автор, год публикации и журнал, продолжительность наблюдения, количество пациентов с соответствующим средним возрастом и ИМТ. Данные, касающиеся следующих PROM, были собраны на исходном уровне и при последнем наблюдении: визуальная аналоговая шкала (ВАШ), шкала активности Тегнера 45 , шкала оценки коленного сустава Лисхольма 46 и Международный комитет документации по коленному суставу (IKDC) 47 . Минимальное клинически значимое различие (МКИД) по ВАШ составило 2,7/10, 10/100 по шкале Лисхольма, 15/100 по шкале IKDC, 0,5/10 по шкале Тегнера 48,49,50 . Также были собраны данные о следующих осложнениях: гипертрофия, неудачи и ревизионные операции.

Оценка риска систематической ошибки и качества рекомендаций

Риск систематической ошибки оценивался в соответствии с рекомендациями Кокрановского справочника по систематическим обзорам вмешательств 51 . Два рецензента (Ф. М. и Х. С.) независимо друг от друга оценили риск систематической ошибки в извлеченных исследованиях. Разногласия решались третьим старшим автором (Н. М.). Рандомизированные контролируемые испытания (РКИ) оценивались с использованием риска систематической ошибки программного обеспечения Review Manager 5.3 (The Nordic Cochrane Collaboration, Копенгаген). Были оценены следующие конечные точки: отбор, обнаружение, производительность, отсев, отчетность и другие отклонения. Не-РКИ оценивались с использованием инструмента «Риск систематической ошибки в нерандомизированных исследованиях вмешательств» (ROBINS-I) 52 . Качество доказательств коллективных исходов оценивали с использованием системы классификации рекомендаций, оценки, разработки и оценки (GRADE) 53,54 .

Методы синтеза

Статистический анализ был выполнен главным автором (Ф. М.) в соответствии с рекомендациями Кокрановского справочника по систематическим обзорам вмешательств 55 . Для описательной статистики использовались среднее значение и стандартное отклонение. Для оценки улучшения по сравнению с исходным уровнем до последнего наблюдения использовалось программное обеспечение SPSS. Была рассчитана средняя разница (MD) с 95% доверительный интервал (ДИ). Парный t-критерий был выполнен со значениями P < 0,05, которые считались статистически значимыми. Для сравнения AMIC с MFx был проведен метаанализ с использованием программного обеспечения Review Manager 5.3 (The Nordic Cochrane Collaboration, Копенгаген). Для описательной статистики использовались средняя разница и стандартное отклонение. Т-тест был выполнен для оценки исходной сопоставимости, при этом значения P > 0,1 считались удовлетворительными. Для непрерывных данных использовался метод обратной дисперсии с мерой эффекта разности средних (MD). Для бинарных данных использовался метод Мантеля-Хензеля с мерой эффекта отношения нечетностей (OR). КИ был установлен на 95% во всем сравнении. Неоднородность оценивали с помощью тестов \(\chi\) 2 и Higgins-I 2 . Если \(\chi \) 2  > 0,05, статистически значимой неоднородности обнаружено не было. Эффект фиксированной модели использовался по умолчанию. Если \(\chi \) 2  < 0,05 и Higgins-I 2  > 60%, была обнаружена высокая неоднородность, и для анализа был использован эффект случайной модели. Общие значения P < 0,05 считались статистически значимыми.

Этическое одобрение

Это исследование соответствует этическим стандартам.

Регистрация и протокол

Настоящий отзыв не зарегистрирован.

Результаты

Выбор исследования

В результате поиска литературы найдено 1211 статей. Из них 301 были исключены из-за дублирования. Еще 890 исследований были исключены, поскольку они не соответствовали критериям приемлемости: не клинические исследования (N = 177), языковые ограничения (N = 5), не фокусировались на коленях (N = 301), не фокусировались на AMIC (N = 407). ). Два исследования не были включены, так как они не сообщали количественные данные по интересующим исходам. Осталось 18 исследований для включения. Результаты литературного поиска представлены на рис. 1.

Рисунок 1

PRISMA Блок-схема поиска литературы.

Полноразмерное изображение

Оценка риска систематической ошибки

Кокрановский инструмент оценки риска систематической ошибки был использован для изучения риска систематической ошибки в РКИ. Учитывая количество ретроспективных исследований, включенных в настоящее исследование, риск систематической ошибки при отборе был умеренным. Немногие авторы проводили ослепление оценщиков, что приводило к умеренному риску систематической ошибки обнаружения. Риск отсева и предвзятости в отчетах был умеренным, как и риск других предубеждений. В заключение, график риска систематической ошибки свидетельствует об умеренном качестве методологической оценки РКИ (рис. 2).

Рисунок 2

Кокрановский инструмент оценки риска систематической ошибки. Риск систематической ошибки при отборе проанализирован при генерации случайной последовательности и сокрытии распределения. Был проанализирован риск систематической ошибки обнаружения в процедуре ослепления во время оценки результатов. Риск систематической ошибки отсева относится к неполным данным об исходах, например, к отсутствию данных об исходах от отсева во время включения в исследование или анализа. Риск систематической ошибки в отчетности связан с выборочной публикацией результатов на основании их статистической или клинической значимости. Если авторы выявляли дополнительный риск систематической ошибки, это рассматривалось как «другая систематическая ошибка». Риск систематической ошибки оценивался в процентах как низкий, высокий или неясный.

Полноразмерное изображение

ROBINS-I применялся для исследования риска систематической ошибки в не-РКИ. Ни одно исследование не показало критического риска систематической ошибки. Учитывая общее приемлемое качество включенных исследований, общий риск систематической ошибки был умеренным (таблица 1).

Таблица 1 ROBINS-I не-RCT.

Полноразмерная таблица

Характеристики исследований и результаты отдельных исследований

Были получены данные 548 пациентов. 33% (180 из 548 пациентов) были женщинами. Среднее последующее наблюдение составило 390,9 ± 26,5 месяцев. Средний возраст составлял 27,0 ± 5,9 лет, а средний ИМТ 27,1 ± 1,3 кг/м 2 . Средний размер дефекта составил 3,2 ± 1,0 см 2 . Общие сведения и демографические данные включенных исследований показаны в таблице 2.

Таблица 2 Общие сведения и исходный уровень пациентов во включенных исследованиях (РКИ: рандомизированное контролируемое исследование).

Полноразмерная таблица

Эффективность AMIC

ВАШ снизился на – 3,9/10 (95% ДИ от − 4,0874 до − 3,7126), шкала активности Тегнера увеличилась на + 0,8/10 (95% ДИ от 0,6595 до 0,9405). Система оценки коленного сустава Lysholm увеличилась на  + 28,9/100 (95% ДИ от 26,8716 до 29,1284), как и IKDC + 33,6/100 (95% ДИ от 32,5800 до 34,6200). Эти результаты показаны более подробно в Таблице 3.

Таблица 3 Улучшение PROM от исходного уровня до последнего наблюдения (FU: последующее наблюдение; MD: средняя разница; CI: доверительный интервал; IKDC: Международный комитет по документации коленного сустава) .

Полноразмерный стол

Осложнения

При последнем осмотре ни у одного пациента не было признаков гипертрофии. 4,3% (9из 210) пациентов, перенесших ревизионную операцию. Частота неудач составила 3,8% (9 из 236).

AMIC по сравнению с MFx

Пять исследований были включены в метаанализ 29,31,37,39,40 . При среднем периоде наблюдения 40,3 месяца группа AMIC продемонстрировала более низкий балл по ВАШ (РС: − 1,01; 95% ДИ от − 1,97 до 0,05) и более высокий IKDC (РС: 11,80; 95% ДИ от 6,65 до 16,94). При среднем периоде наблюдения 43,6 месяца группа AMIC продемонстрировала более низкую частоту ревизий (ОШ: 0,16; 95% ДИ от 0,06 до 0,44). Эти результаты более подробно показаны на рис. 3.

Рисунок 3

Мета-анализ: лесная диаграмма каждого сравнения ( IV обратная дисперсия, OR отношение нечетности, MD средняя разница, MH Mantel-Haenszel, доверительный интервал CI ). Окончательный эффект и относительный доверительный интервал представлены соответственно ромбом и его боковыми концами. Вертикальная линия указывает порог отсутствия эффекта. Эффект и соответствующий доверительный интервал каждого исследования представлены квадратом и горизонтальной линией соответственно.

Полноразмерное изображение

Качество рекомендаций

GRADE обнаружил ограниченный эффект в предполагаемом эффекте, и истинный эффект может существенно отличаться от предполагаемого эффекта. Это связано с низким качеством результатов пересмотра рекомендаций, а также с показателями IKDC и VAS (рис. 4).

Рисунок 4

Общее качество доказательств коллективных исходов в соответствии с подходом GRADE было низким.

Полноразмерное изображение

Обсуждение

Общая интерпретация и клиническое значение

Лечение хондральных дефектов коленного сустава является спорным, с непредсказуемыми результатами. На сегодняшний день ни один из методов не считается окончательным, а резидуальный дефект и рецидив симптомов являются обычным явлением. Согласно основным выводам настоящего исследования, AMIC, по-видимому, эффективен для лечения очаговых хондральных дефектов коленного сустава. Увеличение PROM было больше, чем их MCID 48,49,50 . Кроме того, AMIC показал более высокие значения IKDC, а также более низкое значение VAS и более низкую частоту ревизий по сравнению с MFx. В отличие от других хрящевых процедур, AMIC не требует сбора или расширения какой-либо аутологичной ткани и выполняется за один сеанс хирургического вмешательства. Таким образом, AMIC следует рассматривать у отдельных пациентов с симптоматическим хондральным дефектом коленного сустава.

Предыдущие систематические обзоры оценивали эффективность AMIC. Гао и др. 56 оценили эффективность AMIC в коленном суставе, включая 12 исследований. Они обнаружили снижение ВАШ и улучшение балла по шкале Lysholm в течение первых двух лет наблюдения, но никакого улучшения в период наблюдения от двух до пяти лет 56 . Штайнвакс и др. 57 также выполнил систематический обзор, включающий 12 исследований по AMIC. Оценка Lysholm, IKDC и VAS были улучшены в течение первых двух лет наблюдения, но они продолжали улучшаться через 3 года 57 . Предыдущие систематические обзоры также сравнивали AMIC с другими распространенными хирургическими стратегиями регенерации хряща. Ким и др. 58 сравнили AMIC (13 исследований) с MFx (18 исследований). Они продемонстрировали более высокие значения при оценке IKDC по сравнению с MFx, без различий в шкале Lysholm, шкале активности Tegner и ВАШ боли 58 . Недавний систематический обзор той же исследовательской группы сравнил AMIC с имплантацией аутологичных хондроцитов, индуцированной матрицей (mACI) на колене 59 . Хотя статистических различий между двумя вмешательствами не было, учитывая одноэтапную процедуру, отсутствие забора аутологичного хряща и необходимость расширения хондроцитов в отдельных лабораторных условиях, AMIC может быть предпочтительнее mACI 59 . Другой недавний систематический обзор сравнил AMIC с другими хондральными процедурами, включая только РКИ 60 . В целом, AMIC продемонстрировал эффективность и безопасность при дефектах хряща коленного сустава малого и среднего размера 60 .

Ограничения включенных доказательств

Между исследованиями была очевидна вариабельность. Большинство авторов использовали резорбируемую мембрану из свиного коллагена I/III (Chondroguide®, Geistlich Biomaterials, Wolhusen, Швейцария) 22,24,25,27,29,30,31,34,35,36,38,39,40 . Энеа и др. 32 в 2013 году опубликовано о клиническом применении AMIC с использованием мембраны из полигликолевой кислоты и гиалуроновой кислоты, обогащенной концентратом костного мозга. Та же исследовательская группа в 2015 году опубликовала результаты AMIC с использованием коллагеновой мембраны Biocollagen MeRG® (Bioteck, Виченца, Италия), усиленной концентратом костного мозга. Вариабельность также была обнаружена в технике фиксации мембраны. Большинство авторов фиксировали мембрану фибриновым клеем 22,24,25,27,31,32,33,34,35,36,38,39,40,41 . В дополнение к своим герметизирующим, кровоостанавливающим и адгезивным свойствам фибриновый клей поддерживает миграцию и пролиферацию хондроцитов 61,62,63,64,65,66,67,68,69,70 . Кроме того, фибриновый клей стимулирует фиксацию костно-хрящевого каркаса и регенерацию хряща 71,72,73,74 . Два автора сравнили AMIC, фиксированный с помощью фибринового клея, с шовным материалом 29,37 . Оба автора сообщили о лучших результатах в группе клеевого AMIC. Мембранные швы образуют трещины в суставном хряще, которые могут не заживать и со временем увеличиваться 75,76 . Наложение швов вызывает локальное повреждение хряща, что может привести к боли, замедлению заживления и преждевременному остеоартриту 77 . Большинство авторов выполняли AMIC с использованием миниартротомии или артротомии 22,27,29,31,34,35,36,37,38,39,40,41 . Некоторые авторы 24,25,32,33 использовали артроскопическую технику для выполнения AMIC. Эти различия между исследованиями могут увеличить риск систематической ошибки публикации и снизить надежность настоящего исследования.

Ограничения обзора

Ретроспективный дизайн 55% (10 из 18) включенных исследований представляет собой еще одно важное ограничение настоящего исследования. Учитывая ограниченность данных, доступных для включения, рандомизированные и нерандомизированные исследования не анализировались отдельно. Большинство авторов смешивали пациентов, перенесших хондральные операции на бедренно-большеберцовом и пателло-феморальном суставах, не сообщая результатов отдельно. Более того, большинство авторов приводили данные о пациентах, перенесших комбинированные операции. Описание хирургического подхода, диагноза и реабилитационных протоколов часто было адекватным, как и выбор критериев, показатели результатов и соответствующие сроки оценки. Общие показатели состояния здоровья описывались редко, а процедура оценки исходов часто была необъективной. Чтобы обеспечить высокое качество включенных исследований и связанную с ними достоверность результатов, «серая» литература и статьи, не прошедшие рецензирование, не рассматривались. Это может ограничить количество исследований для включения и может ограничить силу настоящего исследования. Гистопатология новообразованного хряща не сравнивалась в настоящем метаанализе. Характеристики новообразованного хряща при магнитно-резонансной томографии (МРТ) не исследовались. В нескольких исследованиях проанализировано магнитно-резонансное наблюдение системы оценки восстановления хрящевой ткани (MOCART) для оценки качества регенерации хряща. Тем не менее, шкала MOCART не продемонстрировала связи с характеристиками пациента и исходом хирургического вмешательства у пациентов, перенесших хирургическое лечение по поводу дефектов хряща коленного и таранного суставов 78 . Рецензенты (Ф. М. и Х. С.), которые выполняли поиск литературы, извлечение данных, оценку риска систематической ошибки, были основными авторами двух из включенных исследований 39,40 . Это может порождать конфликты. Наконец, дублирующий процесс поиска литературы и извлечения данных не проводился. Эти ограничения негативно повлияли на надежность настоящего исследования. Таким образом, результаты настоящего систематического обзора и метаанализа следует тщательно рассмотреть.

Заключение

AMIC, по-видимому, эффективен для лечения очаговых хрящевых дефектов коленного сустава. Кроме того, AMIC показал более высокий балл IKDC наряду с более низким значением VAS и частотой ревизий по сравнению с MFx. Ограниченное количество и качество включенных исследований ограничивают надежность настоящих результатов и должны интерпретироваться в рамках ограничений настоящего исследования.

Доступность данных

Наборы данных, созданные в ходе и/или проанализированные в ходе текущего исследования, доступны на протяжении всей рукописи.

Ссылки

  1. Curl, W. W. et al. Травмы хряща: обзор 31 516 артроскопий коленного сустава. Артроскопия 13 (4), 456–460. https://doi.org/10.1016/s0749-8063(97)

    -9 (1997).

    КАС Статья пабмед Google ученый

  2. Фигероа, Д. и др. Поражения хряща коленного сустава: Частота и корреляция между результатами артроскопии и магнитного резонанса. Артроскопия 23 (3), 312–315. https://doi.org/10.1016/j.arthro.2006.11.015 (2007 г.).

    Артикул пабмед Google ученый

  3. Наследник, С. и др. Очаговые дефекты хряща в коленном суставе ухудшают качество жизни в той же степени, что и тяжелый остеоартрит: сравнение оценки результатов травмы колена и остеоартрита у 4 категорий пациентов, которым назначена операция на колене. утра. Дж. Спорт Мед. 38 (2), 231–237. https://doi.org/10.1177/0363546509352157 (2010 г.).

    Артикул пабмед Google ученый

  4. Kreuz, P. C. и др. Результаты после микроперелома полнослойных хондральных дефектов в разных отделах коленного сустава. Остеоартроз. Картил. 14 (11), 1119–1125. https://doi.org/10.1016/j.joca.2006.05.003 (2006 г.).

    КАС Статья Google ученый

  5. Силлия, А. Дж. и др. Возвращение в игру спортсменов Национальной футбольной лиги после хондропластики колена. утра. Дж. Спорт Мед. 43 (3), 663–668. https://doi.org/10.1177/0363546514562752 (2015 г.).

    Артикул пабмед Google ученый

  6. Давила Кастродад, И. М., Миз, С. Дж., Верхейм, Э., Макинерни, В. К. и Силлия, А. Дж. Артроскопическое восстановление хондрального дефекта с помощью каркаса внеклеточного матрикса и концентрата аспирата костного мозга. Артроск. Тех. 9 (9), e1241–e1247. https://doi.org/10.1016/j.eats.2020.05.001 (2020 г.).

    Артикул пабмед ПабМед Центральный Google ученый

  7. Филардо Г. и др. Артроскопическая имплантация аутологичных хондроцитов второго поколения в сочетании с костной пластикой для лечения расслаивающего остеохондрита коленного сустава: результаты через 6 лет. Колено 19 (5), 658–663. https://doi.org/10.1016/j.knee.2011.08.007 (2012 г.).

    Артикул пабмед Google ученый

  8. Берто, П. и др. Лечение обширных очагов глубокого остеохондрита коленного сустава с помощью аутологичного матрикс-индуцированного хондрогенеза (AMIC): предварительные результаты у 13 пациентов. Ортоп. травматол. Surg. Рез. 104 (5), 695–700. https://doi.org/10.1016/j.otsr.2018.05.008 (2018 г.).

    Артикул пабмед Google ученый

  9. Гудас Р. и др. Проспективное рандомизированное клиническое исследование мозаичной костно-хрящевой аутологичной трансплантации в сравнении с микропереломами для лечения остеохондральных дефектов коленного сустава у молодых спортсменов. Артроскопия 21 (9), 1066–1075. https://doi.org/10.1016/j.arthro.2005.06.018 (2005 г.).

    Артикул пабмед Google ученый

  10. Richter, D.L., Schenck, R.C. Jr., Wascher, D.C. & Treme, G. Методы восстановления и восстановления суставного хряща коленного сустава: обзор литературы. Спортивное здоровье 8 (2), 153–160. https://doi.org/10.1177/1941738115611350 (2016 г.).

    Артикул пабмед Google ученый

  11. «>

    Смит, Г. Д., Кнутсен, Г. и Ричардсон, Дж. Б. Клинический обзор методов восстановления хряща. Дж. Боун Дж.Т. Surg. бр. 87 (4), 445–449. https://doi.org/10.1302/0301-620X.87B4.15971 (2005 г.).

    КАС Статья Google ученый

  12. Steinwachs, M.R., Guggi, T. & Kreuz, P.C. Методы стимуляции костного мозга. Травмы 39 (Приложение 1), S26-31. https://doi.org/10.1016/j.injury.2008.01.042 (2008 г.).

    Артикул пабмед Google ученый

  13. Вэнь, Х. Дж., Юань, Л. Б., Тан, Х. Б. и Сюй, Ю. К. Микропереломы по сравнению с усиленными методами микропереломов при восстановлении коленного хряща: систематический обзор и метаанализ. Дж. Хирургия коленного сустава. https://doi.org/10.1055/s-0040-1716552 (2020 г.).

    Артикул пабмед Google ученый

  14. «>

    Орт, П., Гао, Л. и Мадри, Х. Микропереломы для восстановления хряща в колене: систематический обзор современной литературы. Хирург коленного сустава. Спортивный травматол. Артроск. 28 (3), 670–706. https://doi.org/10.1007/s00167-019-05359-9 (2020 г.).

    Артикул пабмед Google ученый

  15. Митофер, К., МакАдамс, Т., Уильямс, Р. Дж., Кройц, П. К. и Мандельбаум, Б. Р. Клиническая эффективность метода микропереломов для восстановления суставного хряща в колене: систематический анализ, основанный на фактических данных. утра. Дж. Спорт Мед. 37 (10), 2053–2063. https://doi.org/10.1177/0363546508328414 (2009 г.).

    Артикул пабмед Google ученый

  16. Травлос Г. С. Гистопатология костного мозга. Токсикол. Патол. 34 (5), 566–598. https://doi.org/10.1080/01926230600964706 (2006 г. ).

    Артикул пабмед Google ученый

  17. Травлос Г. С. Нормальная структура, функция и гистология костного мозга. Токсикол. Патол. 34 (5), 548–565. https://doi.org/10.1080/01926230600939856 (2006 г.).

    Артикул пабмед Google ученый

  18. Sebo, Z. L. и др. Ожирение костного мозга: основные и клинические последствия. Эндокр. Ред. 40 (5), 1187–1206. https://doi.org/10.1210/er.2018-00138 (2019 г.).

    Артикул пабмед ПабМед Центральный Google ученый

  19. Чжан, Р., Ма, Дж., Хань, Дж., Чжан, В. и Ма, Дж. Методы лечения поражений хряща и остеоартрита на основе мезенхимальных стволовых клеток. утра. Дж. Пер. Рез. 11 (10), 6275–6289 (2019).

    КАС пабмед ПабМед Центральный Google ученый

  20. «>

    Berthelot, J.M., Le Goff, B. & Maugars, Y. Мезенхимальные стволовые клетки костного мозга при ревматоидном артрите, спондилоартрите и анкилозирующем спондилите: проблемы, а не решения?. Артрит Рез. тер. 21 (1), 239. https://doi.org/10.1186/s13075-019-2014-8 (2019).

    КАС Статья пабмед ПабМед Центральный Google ученый

  21. McGonagle, D., Baboolal, T.G. & Jones, E. Нативные мезенхимальные стволовые клетки, резидентные в суставах, для восстановления хряща при остеоартрите. Нац. Преподобный Ревматол. 13 (12), 719–730. https://doi.org/10.1038/nrrheum.2017.182 (2017 г.).

    КАС Статья пабмед Google ученый

  22. Miyahira, M.K.C. и др. Более крупные хрящевые поражения, обработанные коллагеновой мембраной — аутологичный хондрогенез, индуцированный матриксом, — демонстрируют большее увеличение клинических показателей. Rev. Бюстгальтеры. Ортоп. (Сан-Паулу) 56 (3), 333–339. https://doi.org/10.1055/s-0040-1712493 (2021 г.).

    Артикул Google ученый

  23. Беренс, П. Matrixgekoppelte mikrofrakturierung. Arthroskopie 18 , 193–197 (2005).

    Артикул Google ученый

  24. Скьявоне Панни, А. и др. Хорошие клинические результаты применения метода аутологичного индуцированного матриксом хондрогенеза (Amic) при больших дефектах хряща коленного сустава. Хирургия коленного сустава. Спортивный травматол. Артроск. 26 (4), 1130–1136. https://doi.org/10.1007/s00167-017-4503-0 (2018 г.).

    Артикул пабмед Google ученый

  25. Schagemann, J. et al. Среднесрочные результаты артроскопической AMIC для лечения дефектов суставного хряща в коленном суставе эквивалентны мини-открытым операциям. Арх. Ортоп. травма хирург. 138 (6), 819–825. https://doi.org/10.1007/s00402-018-2887-z (2018 г.).

    КАС Статья пабмед Google ученый

  26. Gotze, C., Nieder, C., Felder, H. & Migliorini, F. AMIC для очагового костно-хрящевого дефекта таранной кости. Life (Базель) 10 , 12. https://doi.org/10.3390/life10120328 (2020).

    Артикул Google ученый

  27. де Джироламо, Л. и др. Аутологичный матрикс-индуцированный хондрогенез (AMIC) и AMIC, усиленный аутологичным концентрированным аспиратом костного мозга (BMAC), обеспечивают стабильные клинические и функциональные улучшения до 9годы наблюдения: результаты рандомизированного контролируемого исследования. Дж. Клин. Мед. 8 , 3. https://doi.org/10.3390/jcm8030392 (2019).

    Артикул Google ученый

  28. «>

    Барк, С. и др. Усовершенствованные методы микропереломов в хирургии хряща коленного сустава: правда или вымысел?. Мир Дж. Ортоп. 5 (4), 444–449. https://doi.org/10.5312/wjo.v5.i4.444 (2014 г.).

    Артикул пабмед ПабМед Центральный Google ученый

  29. Anders, S., Volz, M., Frick, H. & Gellissen, J. Рандомизированное контролируемое исследование, сравнивающее хондрогенез, индуцированный аутологичным матриксом (AMIC(R)) с микропереломами: анализ 1- и 2- годовые данные наблюдения 2 центров. Открытый ортоп. J. 7 , 133–143. https://doi.org/10.2174/1874325001307010133 (2013 г.).

    Артикул пабмед ПабМед Центральный Google ученый

  30. Астур, округ Колумбия и др. Хирургическое лечение хондральных дефектов коленного сустава с использованием коллагеновой мембраны — хондрогенез, индуцированный аутологичной матрицей. Rev. Бюстгальтеры. Ортоп. 53 (6), 733–739. https://doi.org/10.1016/j.rboe.2018.09.005 (2018 г.).

    Артикул пабмед ПабМед Центральный Google ученый

  31. Chung, J. Y. et al. Биомембрана внеклеточного матрикса хряща для улучшения микротрещиноватых дефектов. Хирургия коленного сустава. Спортивный травматол. Артроск. 22 (6), 1249–1259. https://doi.org/10.1007/s00167-013-2716-4 (2014 г.).

    Артикул пабмед Google ученый

  32. Enea, D. и др. Одноэтапная пластика хряща коленного сустава с микропереломом, покрытая резорбируемой матрицей на полимерной основе и концентратом аутологичного костного мозга. Колено 20 (6), 562–569. https://doi.org/10.1016/j.knee.2013.04.003 (2013 г.).

    КАС Статья пабмед Google ученый

  33. «>

    Enea, D. и др. Одноэтапное восстановление хряща коленного сустава: микротрещины, покрытые коллагеном, и аутологичный концентрат костного мозга. Пилотное исследование. Колено 22 (1), 30–35. https://doi.org/10.1016/j.knee.2014.10.003 (2015 г.).

    КАС Статья пабмед Google ученый

  34. Гилле, Дж. и др. Результат индуцированного аутологичным матриксом хондрогенеза (AMIC) в хирургии коленного хряща: данные реестра AMIC. Арх. Ортоп. травма хирург. 133 (1), 87–93. https://doi.org/10.1007/s00402-012-1621-5 (2013 г.).

    КАС Статья пабмед Google ученый

  35. Гудас Р., Мачюлайтис Дж., Стаскунас М. и Смайлис А. Клинические результаты лечения одиночных и множественных дефектов хряща с помощью хондрогенеза, индуцированного аутологичным матриксом. Дж. Ортоп. Surg. (Гонконг) 27 (2), 230949

    51011. https://doi.org/10.1177/230949

    51011 (2019).

    Артикул Google ученый

  36. Ланер, М. и др. Хирургия хряща у пациентов с избыточной массой тела: клинические и МРТ-результаты после процедуры хондрогенеза, индуцированного аутологичным матриксом. Биомед. Рез. Междунар. 2018 , 6363245. https://doi.org/10.1155/2018/6363245 (2018).

    Артикул пабмед ПабМед Центральный Google ученый

  37. Volz, M., Schaumburger, J., Frick, H., Grifka, J. & Anders, S. Рандомизированное контролируемое исследование, демонстрирующее устойчивое преимущество аутологичного матрикс-индуцированного хондрогенеза по сравнению с микропереломами через пять лет. Междунар. Ортоп. 41 (4), 797–804. https://doi.org/10.1007/s00264-016-3391-0 (2017 г.).

    Артикул пабмед Google ученый

  38. «>

    Жиль, Дж. и др. Хондрогенез, индуцированный аутологичным матриксом, для лечения очаговых дефектов хряща в коленном суставе: последующее исследование. Ортоп. Дж. Спорт Мед. 9 (2), 2325967120981872. https://doi.org/10.1177/2325967120981872 (2021).

    Артикул пабмед ПабМед Центральный Google ученый

  39. Мильорини, Ф. и др. Аутологичный матрикс-индуцированный хондрогенез (AMIC) и микропереломы при фокальных хондральных дефектах коленного сустава: среднесрочное сравнительное исследование. Life (Базель) 11 , 3. https://doi.org/10.3390/life11030183 (2021).

    Артикул Google ученый

  40. Мильорини, Ф. и др. Лечение дефектов хряща надколенника с помощью хондрогенеза, индуцированного аутологичным матриксом (AMIC), по сравнению с микропереломами: четырехлетнее клиническое исследование. Life (Базель) 11 , 2. https://doi.org/10.3390/life11020141 (2021).

    Артикул Google ученый

  41. Tradati, D. и др. Техника хондрогенеза, индуцированного AMIC-аутологичным матриксом, при лечении дефектов хряща надколенника: ретроспективное исследование со среднесрочным наблюдением. Дж. Клин. Мед. 9 , 4. https://doi.org/10.3390/jcm

    84 (2020).

    Артикул Google ученый

  42. Waltenspul, M., Suter, C., Ackermann, J., Kuhne, N. & Fucentese, S. F. Аутологичный матрикс-индуцированный хондрогенез (AMIC) для изолированных поражений ретропателлярного хряща: результат после наблюдения минимум 2 годы. Хрящ https://doi.org/10.1177/19476035211021908 (2021).

    Артикул пабмед ПабМед Центральный Google ученый

  43. «>

    Howick J CI, Glasziou P, Greenhalgh T, Carl Heneghan, Liberati A, Moschetti I, Phillips B, Thornton H, Goddard O, Hodgkinson M (2011) Oxford CEBM Levels of Evidence. Оксфордский центр доказательной медицины Доступно по ссылке https://www.cebm.net/indexaspx?o=5653

  44. Page, M.J. и др. Заявление PRISMA 2020: обновленное руководство по составлению отчетов о систематических обзорах. БМЖ 372 , №71. https://doi.org/10.1136/bmj.n71 (2021 г.).

    Артикул пабмед ПабМед Центральный Google ученый

  45. Бриггс, К. К. и др. Надежность, достоверность и отзывчивость шкалы Лисхольма и шкалы активности Тегнера при травмах передней крестообразной связки колена: 25 лет спустя. 902:16 утра. Дж. Спорт Мед. 37 (5), 890–897. https://doi.org/10.1177/0363546508330143 (2009 г.).

    Артикул пабмед Google ученый

  46. «>

    Lysholm, J. & Gillquist, J. Оценка результатов операции на связках коленного сустава с особым акцентом на использование оценочной шкалы. утра. Дж. Спорт Мед. 10 (3), 150–154. https://doi.org/10.1177/036354658201000306 (1982 г.).

    КАС Статья пабмед Google ученый

  47. Хиггинс Л.Д., Тейлор М.К., Парк Д., Годадра Н., Маршан М., Пьетробон Р., Кук С., Международная документация по коленному суставу C. Надежность и валидность субъективной формы коленного сустава Международного комитета по документации коленного сустава (IKDC). Совместная кость, позвоночник 74 (6), 594–599. https://doi.org/10.1016/j.jbspin.2007.01.036 (2007 г.).

    Артикул Google ученый

  48. Мостафаи, Н. и др. Чувствительность персидской версии оценки результатов травмы колена и остеоартрита и шкалы активности Тегнера у спортсменов с реконструкцией передней крестообразной связки после физиотерапевтического лечения. Физиотерапевт. Теория Практика. 36 (9), 1019–1026. https://doi.org/10.1080/09593985.2018.1548672 (2020 г.).

    Артикул пабмед Google ученый

  49. Джонс, К. Дж., Келли, Б. В., Арши, А., Макаллистер, Д. Р. и Фабрикант, П. Д. Сравнительная эффективность восстановления хряща с учетом минимальной клинически значимой разницы. утра. Дж. Спорт Мед. 47 (13), 3284–3293. https://doi.org/10.1177/0363546518824552 (2019 г.)).

    Артикул пабмед Google ученый

  50. Агарвалла, А. и др. Возвращение к спорту после изолированной клиновидной остеотомии дистального отдела бедренной кости с боковым отверстием. Хрящ 13 (1), 846S-852S. https://doi.org/10.1177/1947603520924775 (2021 г.).

    КАС Статья пабмед Google ученый

  51. «>

    Кампстон, М. и др. Обновленное руководство по надежным систематическим обзорам: новое издание Кокрейновского справочника по систематическим обзорам вмешательств. Кокрановская система базы данных. Ред. 10 , 142. https://doi.org/10.1002/14651858.ED000142 (2019).

    Артикул Google ученый

  52. Стерн, Дж. А. и др. ROBINS-I: инструмент для оценки риска систематической ошибки в нерандомизированных исследованиях вмешательств. БМЖ 355 , i4919. https://doi.org/10.1136/bmj.i4919 (2016 г.).

    Артикул пабмед ПабМед Центральный Google ученый

  53. Brozek, J. L. et al. Оценка качества доказательств и силы рекомендаций в руководствах по клинической практике. Часть 1 из 3. Обзор подхода GRADE и оценки качества доказательств в отношении вмешательств. Аллергия 64 (5), 669–677. https://doi.org/10.1111/j.1398-9995.2009.01973.х (2009).

    КАС Статья пабмед Google ученый

  54. Аткинс, Д. и др. Оценка качества доказательств и силы рекомендаций. BMJ 328 (7454), 1490. https://doi.org/10.1136/bmj.328.7454.1490 (2004).

    Артикул пабмед Google ученый

  55. Higgins JPT TJ, Chandler J, Cumpston M, Li T, Page MJ, Welch VA . Кокрановский справочник по систематическим обзорам вмешательств, версия 6.2. Cochrane 2021. Доступно на сайте www.training.cochrane.org/handbook. По состоянию на февраль 2022 г.

  56. Гао, Л., Орт, П., Куккиарини, М. и Мадри, Х. Хондрогенез, индуцированный аутологичным матриксом: систематический обзор клинических данных. утра. Дж. Спорт Мед. 47 (1), 222–231. https://doi.org/10.1177/0363546517740575 (2019 г. ).

    Артикул пабмед Google ученый

  57. Steinwachs, M. R. et al. Систематический обзор и метаанализ клинических данных об использовании хондрогенеза, индуцированного аутологичным матриксом, в коленном суставе. Хрящ https://doi.org/10.1177/1947603519870846 (2019).

    Артикул пабмед ПабМед Центральный Google ученый

  58. Ким, Дж. Х., Хео, Дж. В. и Ли, Д. Х. Клинические и радиологические результаты после хондрогенеза, индуцированного аутологичным матриксом, по сравнению с микропереломами коленного сустава: систематический обзор и метаанализ с минимальным 2-летним наблюдением. Ортоп. Дж. Спорт Мед. 8 (11), 2325967120959280. https://doi.org/10.1177/2325967120959280 (2020).

    Артикул пабмед ПабМед Центральный Google ученый

  59. «>

    Мильорини, Ф. и др. Индуцированная матриксом имплантация аутологичных хондроцитов по сравнению с индуцированным аутологичным матриксом хондрогенезом при хондральных дефектах таранной кости: систематический обзор. Бр. Мед. Бык. 138 (1), 144–154. https://doi.org/10.1093/bmb/ldab008 (2021 г.).

    Артикул пабмед Google ученый

  60. Karpinski, K., Haner, M., Bierke, S. & Petersen, W. Хондрогенез, индуцированный матриксом, является действительным и безопасным вариантом восстановления хряща для дефектов хряща коленного сустава малого и среднего размера: Систематический обзор. Хирург коленного сустава. Спортивный травматол. Артроск. https://doi.org/10.1007/s00167-021-06513-y (2021).

    Артикул пабмед Google ученый

  61. Наир, М. А., Шайк, К. В., Коккилигадда, А. и Горрела, Х. Тканеинженерная реконструкция челюстно-лицевого скелетного дефекта с помощью 3D-печатного каркаса из бета-трикальцийфосфата, связанного факторами роста и фибриновым клеем, имплантированного аутологичного костномозгового мезенхимального стволовые клетки. Ж. Мед. Лайф 13 (3), 418–425. https://doi.org/10.25122/jml-2020-0044 (2020 г.).

    Артикул пабмед ПабМед Центральный Google ученый

  62. Сангани-Кераи, А. и др. Разработка нового аутологичного кровяного клея, направленного на улучшение остеоинтеграции в области контакта кость-имплантат. Рез. сустава кости. 9 (7), 402–411. https://doi.org/10.1302/2046-3758.97.BJR-2019-0073.R3 (2020 г.).

    Артикул пабмед ПабМед Центральный Google ученый

  63. Cassaro, C.V. и др. Биополимер фибрина в качестве подложки для лечения дефектов кости у крыс. Дж. Веном. Аним. Токсины вкл. Троп. Дис. 25 , е201

  64. . https://doi.org/10.1590/1678-9199-JVATITD-2019-0027 (2019 г.).

    КАС Статья пабмед ПабМед Центральный Google ученый

  65. «>

    Моралес-Конде, С., Балла, А., Аларкон, И. и Санчес-Рамирес, М. Минимально инвазивная пластика вентральной грыжи с помощью одной трети скоб и фибринового клея: меньше боли и такая же частота рецидивов. Минерва Чир. 75 (5), 292–297. https://doi.org/10.23736/S0026-4733.20.08468-0 (2020 г.).

    Артикул пабмед Google ученый

  66. Вонг, А. И., Макдональд, А., Джонс, Б. и Берковиц, Д. Пластырь и клей: Новый метод восстановления бронхопищеводных свищей и удаления бронхолитов с помощью стента и фибринового клея. Дж. Бронхол. Интерв. Пульмонол. https://doi.org/10.1097/LBR.0000000000000732 (2020).

    Артикул Google ученый

  67. Карп, Дж. М. и др. Опосредованная тромбином миграция остеогенных клеток. Кость 37 (3), 337–348. https://doi.org/10.1016/j.bone.2005.04.022 (2005 г.).

    КАС Статья пабмед Google ученый

  68. Brown, L. F. и др. Миграция фибробластов в матрицах фибринового геля. утра. Дж. Патол. 142 (1), 273–283 (1993).

    КАС пабмед ПабМед Центральный Google ученый

  69. Тани, К. и др. Тромбин усиливает пролиферацию фибробластов легких при блеомицин-индуцированном легочном фиброзе. утра. Дж. Дыхание. Ячейка Мол. биол. 5 (1), 34–40. https://doi.org/10.1165/ajrcmb/5.1.34 (1991).

    КАС Статья пабмед Google ученый

  70. Chinni, C. и др. Тромбин, фактор выживания культивируемых миобластов. J. Biol. хим. 274 (14), 9169–9174. https://doi.org/10.1074/jbc.274.14.9169 (1999).

    КАС Статья пабмед Google ученый

  71. «>

    Pagel, C. N. и др. Ингибирование апоптоза остеобластов тромбином. Кость 33 (4), 733–743. https://doi.org/10.1016/s8756-3282(03)00209-6 (2003 г.).

    КАС Статья пабмед Google ученый

  72. Филардо Г. и др. Фибриновый клей улучшает фиксацию костно-хрящевого каркаса: исследование трупного колена человека, подвергавшегося непрерывным пассивным движениям. Остеоартроз. Картил. 22 (4), 557–565 (2014).

    КАС Статья Google ученый

  73. Хомминга Г. Н., Бума П., Кут Х. В., ван дер Краан П. М. и ван ден Берг В. Б. Поведение хондроцитов в фибриновом клее in vitro. Акта Ортоп. Сканд. 64 (4), 441–445. https://doi.org/10.3109/17453679308993663 (1993 г.).

    КАС Статья пабмед Google ученый

  74. «>

    Сингх, К., Мойер, Х., Уильямс, Дж. К., Шварц, З. и Боян, Б. Д. Фибриновый клей: каркас для клеточной терапии дефекта критического размера. Энн. Пласт. Surg. 66 (3), 301–305. https://doi.org/10.1097/SAP.0b013e3181fc0507 (2011 г.).

    КАС Статья пабмед Google ученый

  75. Чжэн, М. Х. и др. Индуцированная матриксом имплантация аутологичных хондроцитов (MACI): биологическая и гистологическая оценка. Ткань Eng. 13 (4), 737–746. https://doi.org/10.1089/ten.2006.0246 (2007 г.).

    КАС Статья пабмед Google ученый

  76. Уокер, Э. А., Вернер, А., Фланнери, К. Р. и Арчер, К. В. Клеточные реакции эмбрионального гиалинового хряща на экспериментальное ранение in vitro. Дж. Ортоп. Рез. 18 (1), 25–34. https://doi.org/10.1002/jor.1100180105 (2000 г.).

    КАС Статья пабмед Google ученый

  77. «>

    Hunziker, E.B. & Quinn, T.M. Хирургическое удаление суставного хряща приводит к потере хондроцитов из хряща, граничащего с краем раны. Дж. Боун Дж.Т. Surg. Являюсь. 85 , 85–92. https://doi.org/10.2106/00004623-200300002-00011 (2003 г.).

    Артикул Google ученый

  78. Hunziker, E.B. & Stahli, A. Хирургическое наложение швов на суставной хрящ вызывает остеоартрозоподобные изменения. Остеоартроз. Картил. 16 (9), 1067–1073. https://doi.org/10.1016/j.joca.2008.01.009 (2008 г.).

    КАС Статья Google ученый

  79. Migliorini, F. et al. Надежность оценки MOCART: систематический обзор. Дж. Ортоп. травматол. 22 (1), 39. https://doi.org/10.1186/s10195-021-00603-w (2021).

    Артикул пабмед ПабМед Центральный Google ученый

Ссылки на скачивание

Финансирование

Финансирование в открытом доступе организовано и организовано Projekt DEAL. Авторы не получали никакой финансовой поддержки для исследования, авторства и/или публикации этой статьи.

Информация о авторе

Авторы и принадлежности

  1. Отдел ортопедических, травмы и реконструктивная хирургия, Университетская больница RWTH, Pauwelsstraße 30, 52074, Aachen, Germany

    FILIPPO MigliorIni, Alice Barani, AACHEN, Germany

    FILIPOMARINI, ACHINCINI, AACHEN, GEMARY

    .

  2. Кафедра ортопедической и травматологической хирургии, Eifelklinik St. Brigida, 52152, Зиммерат, Германия

    Андреас Белл

  3. Кафедра медицины, хирургии и стоматологии, Университет Салерно, 84081, Баронисси, ЮАР, Италия

    Никола Маффулли

  4. Факультет фармации и биоинженерии, Медицинский факультет Кильского университета, ST4 7QB, Сток-он-Трент, Англия Стоматология, Центр спортивной и лечебной физкультуры, Больница Майл Энд, E1 4DG, Лондон, Англия

    Никола Маффулли

Авторы

  1. Филиппо Мильорини

    Посмотреть публикации авторов

    Вы также можете искать этого автора в PubMed Google Scholar

  2. Никола Маффулли

    Просмотр публикаций автора

    Вы также можете искать этого автора в PubMed Google Scholar

  3. Alice Baroncini

    Просмотр публикаций автора

    Вы также можете искать этого автора в PubMed Google Scholar

  4. Andreas Bell

    Просмотр публикаций автора

    Вы также можете искать этого автора в PubMed Google Академия

  5. Frank Hildebrand

    Просмотр публикаций автора

    Вы также можете искать этого автора в PubMed Google Scholar

  6. Hanno Schenker

    Просмотр публикаций автора

    Вы также можете искать этого автора в PubMed Google Scholar

Вклады

FM: поиск литературы, извлечение данных, оценка риска смещения, концепция и дизайн, составление проекта, окончательное утверждение; N. M.: надзор, доработка, окончательное утверждение; FH: написание, окончательное утверждение; A.B.: поиск литературы, извлечение данных, оценка риска систематической ошибки, окончательное утверждение; A.B.: надзор, окончательное утверждение; HS: составление проекта, окончательное утверждение. Все авторы согласились с публикацией окончательной версии и согласны нести ответственность за все аспекты работы.

Автор, ответственный за переписку

Переписка с Филиппо Мильорини.

Заявление об этике

Конкурирующие интересы

Авторы не заявляют об отсутствии конкурирующих интересов.

Дополнительная информация

Примечание издателя

Springer Nature остается нейтральной в отношении юрисдикционных претензий в опубликованных картах и ​​институциональной принадлежности.

Дополнительная информация

Дополнительная информация.

Права и разрешения

Открытый доступ Эта статья находится под лицензией Creative Commons Attribution 4. 0 International License, которая разрешает использование, совместное использование, адаптацию, распространение и воспроизведение на любом носителе или в любом формате, при условии, что вы укажете соответствующую ссылку на первоначальный автор(ы) и источник, предоставьте ссылку на лицензию Creative Commons и укажите, были ли внесены изменения. Изображения или другие сторонние материалы в этой статье включены в лицензию Creative Commons на статью, если иное не указано в кредитной строке материала. Если материал не включен в лицензию Creative Commons статьи, а ваше предполагаемое использование не разрешено законом или выходит за рамки разрешенного использования, вам необходимо получить разрешение непосредственно от правообладателя. Чтобы просмотреть копию этой лицензии, посетите http://creativecommons.org/licenses/by/4.0/.

Перепечатка и разрешения

Об этой статье

Комментарии

Отправляя комментарий, вы соглашаетесь соблюдать наши Условия и правила сообщества. Если вы обнаружите что-то оскорбительное или не соответствующее нашим условиям или правилам, отметьте это как неприемлемое.

Алгебраическая и геометрическая кратность собственных значений

Марко Табога, доктор философии

Алгебраическая кратность собственного значения — это количество раз, которое оно встречается как корень характеристического многочлена (т. е. многочлена, корни которого собственные значения матрицы).

Геометрическая кратность собственного значения есть размерность линейной пространство связанных с ним собственных векторов (т. е. его собственное пространство).

В этой лекции мы даем строгие определения двух концепций алгебраическая и геометрическая кратность, и мы доказываем некоторые полезные факты о их.

Ищете калькулятор геометрической кратности или пошаговый учебник о том, как рассчитать геометрическую кратность? Перейдите по этой ссылке.

Table of contents

  1. Algebraic multiplicity

  2. Geometric multiplicity

  3. Relationship between algebraic and geometric multiplicity

  4. Defective eigenvalues ​​

  5. Solved exercises

    1. Exercise 1

    2. Упражнение 2

Алгебраическая кратность

Начнем с определения.

Определение Позволять быть матрица. Обозначим через в возможно повторяется собственные значения , которые решают характеристику уравнениеМы сказать, что собственное значение имеет алгебраическую кратность тогда и только тогда, когда их не больше и не меньше решения характеристического уравнения, равные .

Давайте посмотрим на некоторые примеры.

Пример Рассмотрим матрица характеристический полином это корни многочлена, то есть решения Таким образом, имеет два различных собственных значения. Их алгебраические кратности равны потому что они не повторяются.

Пример Определите матрица Его характеристический полином это корни многочлена, то есть решения Таким образом, имеет одно повторяющееся собственное значение, алгебраическая кратность которого равна

Геометрическая кратность

Напомним, что каждое собственное значение связано с линейное пространство собственных векторов, называется собственным пространством.

Определение Позволять быть матрица. Позволять быть одним из собственных значений и обозначим его ассоциированное собственное пространство через . Размер называется геометрической кратностью собственного значения .

Давайте теперь сделаем несколько примеров.

Определение Рассмотрим матрица характеристический полином это корни многочлена являются собственные векторы, связанные с являются векторами что решить уравнениеили последнее уравнение подразумевает чтоПоэтому, собственное пространство это линейное пространство, которое содержит все векторы принадлежащий формагде может быть любым скаляром. Таким образом, собственное пространство генерируется не замужем векторСледовательно, он имеет измерение . Как следствие, геометрическая кратность является .

Пример Рассмотрим матрица характеристический полином остров его корни Таким образом, существует повторяющееся собственное значение () с алгебраической кратностью, равной 2. Его ассоциированные собственные векторы решать в уравнениеили уравнение выполняется для и любое значение . Как следствие, собственное пространство это линейное пространство, которое содержит все векторы принадлежащий формагде может быть любым скаляром. Поскольку собственное пространство генерируется одним векторэто имеет измерение . Как следствие, геометрическая кратность равно 1, что меньше его алгебраической кратности, равной 2.

Пример Определите матрица характеристический полином остров его корни Таким образом, существует повторяющееся собственное значение () с алгебраической кратностью, равной 2. Его ассоциированные собственные векторы решать в уравнениеили уравнение выполняется для любого значения а также . Как следствие, собственное пространство это линейное пространство, которое содержит все векторы принадлежащий формагде а также являются скалярами, которые могут быть выбраны произвольно. Таким образом, собственное пространство создается двумя линейно независимый векторыСледовательно, он имеет измерение . Как следствие, геометрическая кратность равно 2, что равно его алгебраической кратности.

Вывод из предыдущих примеров состоит в том, что алгебраическая и геометрическая кратность собственного значения не обязательно совпадают.

Связь между алгебраической и геометрической кратностью

Следующее предложение устанавливает важное свойство кратностей.

Предложение Позволять быть матрица. Позволять быть одним из собственных значений . Тогда геометрическая кратность меньше или равно его алгебраической кратности.

Доказательство

Предположим, что геометрическая кратность равно , так что есть линейно независимые собственные векторы связаны с . Произвольно выбрать векторы , все имеющие измерение и такой, что столбцы векторов линейно независимы. Определите матрица За Любые , обозначать через вектор, который решает, какой гарантированно существует, потому что является полноранговым (его столбцы линейно независимы). Определите матрица и обозначать через его верхний блокировать и по ниже блок:обозначить по в единичная матрица. Для любого скаляра , у нас есть что так как является полноправным и, как следствие, его определитель ненулевое, мы можем написатьгде в ногу мы использовали результат о определитель блочные матрицы. Собственные значения решить характеристическое уравнение или же, эквивалентно, уравнениеЭто уравнение имеет корень это повторяется по крайней мере раз. Следовательно, алгебраическая кратность не меньше его геометрической кратности . Может быть больше, если также является корнем

Дефектные собственные значения

Когда геометрическая кратность повторяющегося собственного значения строго меньше его алгебраической кратности, то это собственное значение называется дефектный .

Собственное значение, которое не повторяется, имеет связанный с ним собственный вектор, равный отличное от нуля. Следовательно, размерность его собственного пространства равна 1, его геометрическая кратность равна 1 и равна его алгебраической множественность. Таким образом, неповторяющееся собственное значение также является недефектным.

Решенные упражнения

Ниже вы можете найти несколько упражнений с поясненными решениями.

Упражнение 1

Найдите, является ли матрица имеет любые дефектные собственные значения.

Решение

Характеристический многочлен остров его корни Таким образом, нет повторяющихся собственных значений и, как следствие, дефектных собственные значения.

Упражнение 2

Определить

Определить имеет дефектные собственные значения.

Решение

Характеристический полином это здесь в ногу мы использовали Лаплас расширение по третьему ряду. Корни многочлена Таким образом, существует повторяющееся собственное значение () с алгебраической кратностью, равной 2. Его ассоциированные собственные векторы решать в уравнениеили уравнение выполняется для любого значения а также . Как следствие, собственное пространство это линейное пространство, которое содержит все векторы принадлежащий формагде скаляр может быть выбран произвольно.

alexxlab

Добавить комментарий

Ваш адрес email не будет опубликован. Обязательные поля помечены *